Download as pdf or txt
Download as pdf or txt
You are on page 1of 41

VISIONIAS

www.visionias.in
ANSWERS & EXPLANATION
GENERAL STUIDES (P) 2020 – ABHYAAS TEST – 1 (3000)

Q 1.D
• Positive liberty is the possibility of acting — or the fact of acting — in such a way as to take control of
one's life and realize one's fundamental purposes.
• Negative liberty is the absence of external obstacles, barriers or constraints.
• While negative liberty is usually attributed to individual agents, positive liberty is sometimes attributed to
collectivities, or to individuals considered primarily as members of given collectivities. In its political
form, positive freedom has often been thought of as necessarily achieved through a collectivity.
• Negative Liberty in the Indian Constitution:
o Article 14 states that “State shall not deny to any person equality before the law or the equal
protection of the laws within the territory of India.” In this article, the constitution has put an
obligation overstate to ensure equality before the law and equal protection of the law. Thus, it is
negative in nature.
o Article 21: Protection of life and personal liberty "No person shall be deprived of his life or personal
liberty except according to the procedure established by law". This again puts an obligation on states
to not deprive a person of his/her life and personal liberty. The event could be attributed to negative
liberty.
o Article 22(1) Protection against arrest and detention in certain cases: No person who is arrested shall
be detained in custody without being informed, as soon as may be, of the grounds for such arrest nor
shall he be denied the right to consult, and to be defended by, a legal practitioner of his choice
• Positive rights or liberty are those which confer certain privileges on the person or community. For
example, Right to religious freedom (Article 25-28) and cultural and educational right (Article 29-30).
These rights confer certain special provisions for protection and upliftment of minorities. Thus, these are
called positive rights.
o Article 25 (1) Freedom of conscience and free profession, practice and propagation of
religion: Subject to public order, morality and health and to the other provisions of this Part, all
persons are equally entitled to freedom of conscience and the right freely to profess, practice and
propagate religion. This statement confers privilege upon a person to carry out his religious affairs,
thus this could be seen as a positive concept of rights.
o Article 16: Equality of opportunity in matters of public employment : (1) There shall be equality of
opportunity for all citizens in matters relating to employment or appointment to any office under the
State
Q 2.D
• Domestic Gas Sources: The domestic gas in the country is being supplied from the oil & gas fields located
in western and southeastern areas viz. Hazira basin, Mumbai offshore & KG basin as well as the North
East Region (Assam & Tripura).
• Recently the International Energy Agency (IEA) has slammed India's natural gas pricing policy, saying
linking domestic production to very low global reference prices has reduced incentives for producers to
raise supplies. Currently, the price of domestically produced natural gas is fixed by a formula that
averages out rates in gas surplus nations such as Russia and the US. Hence statement 1 is not
correct.
• Natural Gas in India does not fall within the purview of GST. When the Goods and Services Tax was
introduced on July 1, 2017, amalgamating 17 central and state levies, five commodities namely crude oil,
natural gas, petrol, diesel, and aviation turbine fuel were kept out of its purview given the revenue
dependence of state governments on this sector. Currently, natural gas is taxed under the VAT regime
with VAT ranging from 3 percent to 20 percent across states. Hence statement 2 is not correct.
• The IEA said for the share of environment-friendly fuel to rise, the government needs to ensure gas is
treated on a level playing field with other fuels for taxation and is included under the Goods and Services
1 www.visionias.in ©Vision IAS
Tax (GST). Bringing natural gas under the ambit of goods and services tax (GST) will not only make
transport of the fuel across the country more efficient but also facilitate setting up of the much-discussed
domestic gas trading hub.
• With stagnant domestic output, India meets half of its gas needs through imports. Russia is one of
the major countries to begin supplying long-term LNG to India after the United States, Australia, and
Qatar. Hence statement 3 is not correct. In order to meet the gas demand, Liquefied Natural Gas (LNG)
is imported through Open General License (OGL) in the country and it is imported by the gas marketer
under various Long Term, Medium Term, and Spot contracts. The price and utilization of imported LNG
are mutually decided by buyers and sellers. The price of the Natural Gas according to this formula
currently is USD 3.23 per million British thermal units, half of what India pays for import of liquefied
natural gas (LNG).
• National Gas Grid: At present, there are about 16800 km long Natural Gas pipeline network which is
operational in the country. In order to make available natural gas across the country, it has been envisaged
to develop an additional about 14,300 km pipelines to complete the National Gas Grid. To promote the
development of City Gas Distribution Network, the Government has accorded the priority in domestic gas
allocation to PNG (Domestic) and CNG (Transport) segments. It has been decided to meet 100% gas
requirement of CNG (T) and PNG(D) segments through the supply of domestic gas which is cheaper than
imported gas. The India National Gas Grid Technical Assistance program stems from an agreement in
September between PNGRB and the United States Trade Development Agency (USTDA).

Q 3.C
• The Wetlands (Conservation and Management) Rules, 2017 shall apply to the following wetlands or
wetlands complexes, namely:—
o wetlands categorised as 'wetlands of international importance' under the Ramsar Convention;
o wetlands as notified by the Central Government, State Government and Union Territory
Administration:
• Provided that these rules shall not apply to the wetlands falling in areas covered under the Indian
Forest Act, 1927, the Wildlife (Protection) Act, 1972, the Forest (Conservation) Act, 1980, the State
Forest Acts, and the Coastal Regulation Zone Notification, 2011 as amended from time to time.
• All wetlands, irrespective of their location, size, ownership, biodiversity, or ecosystem services values,
can be notified under the Wetlands Rules, except:
o River channels;
o Paddy fields;
o Human-made waterbodies specifically constructed for drinking water purposes;
o Human-made waterbodies specifically constructed for aquaculture purposes;
o Human-made waterbodies specifically constructed for salt production purposes;
o Human-made waterbodies specifically constructed for recreation purposes;
o Human-made waterbodies specifically constructed for irrigation purposes;
o Wetlands falling within areas covered under the Indian Forest Act, 1927; Forest (Conservation) Act,
1980; State Forest Acts and amendments thereof;
o Wetlands falling within areas covered under the Wildlife (Protection) Act, 1972 and amendments
thereof;
o Wetlands falling within areas covered under the Coastal Regulation Zone Notification, 2011 and
amendments thereof.
Q 4.C
• In order to promote and encourage the use of biogas as clean cooking fuel, the earlier scheme, National
Biogas and Manure Management Programme (NBMMP) was modified and is now being implemented as
New National Biogas and Organic Manure Programme (NNBOMP) from 1st April 2018. The Rural
Development Departments of the State Governments or UT Administrations have been designated as the
main Programme Implementing Agencies.
• It is a Central Sector Scheme and is being implemented with the objective:
o To provide clean cooking fuel for kitchens, lighting and meeting other thermal and small power
needs of farmers/dairy farmers /users including individual households and to improve organic
manure system (not eliminate chemical fertilizer) based on bio-slurry from biogas plants in rural
and semi-urban areas by setting up of small size biogas plants of 1 to 25 Cubic Metre capacity. Hence
statements 1 and 2 are correct and statement 3 is not correct.
o To mitigate drudgery of women and time saving for them for other livelihood activities and reduce
pressure on forests and accentuate social benefits;

2 www.visionias.in ©Vision IAS


o To improve sanitation in rural and semi-urban areas including linking sanitary toilets with cattle dung
biogas plants;
o To provide biogas plant produced slurry (liquid / semi-solid or dried) as an organic enriched bio-
manure to help reduce use of chemical fertilizers such as urea, linking biogas slurry with enrichment
units such as vermicomposting, Phosphate Rich Organic Manure (PROM) plants and other organic
enrichment facilities as a source of value addition to bio gas plant slurry.
o To meet ‘lifeline energy’ needs for clean cooking as envisaged in “Integrated Energy Policy” of NITI
Aayog (erstwhile Planning Commission).
o To help in combating and reduction in causes of climate change by preventing emissions of
Green House Gases ( GHGs) such as carbon dioxide. Hence statement 4 is correct.
Q 5.D
• The term RNA interference (RNAi) was coined to describe a cellular mechanism that uses the gene's own
DNA sequence of the gene to turn it off, a process that researchers call silencing. In a wide variety of
organisms, including animals, plants, and fungi, RNAi is triggered by double-stranded RNA
(dsRNA). Hence option (d) is the correct answer.
• During RNAi, long dsRNA is cut or "diced" into small fragments ~21 nucleotides long by an enzyme
called "Dicer". These small fragments referred to as small interfering RNAs (siRNA), bind to proteins
from a special family: the Argonaute proteins.
• RNAi is widely used by researchers to silence genes in order to learn something about their function.
siRNAs can be designed to match any gene, can be manufactured cheaply, and can be readily
administered to cells. One can now order commercially synthesized siRNAs to silence virtually any gene
in a human or other organism's cell, dramatically accelerating the pace of biomedical research.
Furthermore, the ability to turn off the expression of a single gene makes RNAi an appealing therapeutic
approach to treat infectious diseases or genetic disorders, such as those that result from the inappropriate
and undesirable activity of a gene, as in many cancers and neurodegenerative diseases. There are currently
several clinical trials testing the safety and effectiveness of siRNA drugs.

Q 6.C
• The period that followed Rig Vedic Age is known as the Later Vedic Age. It spanned from c. 1000 – c.
600 BCE.
• Statement 1 is not correct: The Vedic communities had developed neither a taxation system nor a
professional army. There did not exist collectors of taxes apart from the kinsmen of the prince.
• Statement 2 is correct: The peasantry in the later Vedic times was called bala (force). The army to
protect the asvamedha horse comprised both the kshatriyas and the vis. Although the nobles and warriors
ruled over heir peasant kinsmen, they had to depend upon peasant militia for fighting against
enemies and they could not grant land without the consent of the peasantry. All this placed them in a
difficult position and could not sharpen the distinctions between the rulers and the ruled.
• Statement 3 is not correct: The mode of worship changed considerably in the Later Vedic period.
Prayers were recited, but they ceased to be the dominant mode of
worship. Instead, sacrifices became far more important. They had both a public and a private
character. Public sacrifices involved the king and the whole of the community while private
sacrifices were performed by individuals in their houses because in this period the people led a settled life
and maintained well-established households. Sacrifices involved the killing of animals on a large scale
and especially the destruction of cattle wealth.
• During the later Vedic period, the emergence of Rajan as the wielder of supreme political power involved
his distancing himself from those closest to him – his kinsmen. This distancing was emphasized in
ritualized contests such as the chariot race in the vajpeya sacrifice (meant to re-establish a king’s
supremacy over his people), and the cattle raid and game of dicing in the rajasuya sacrifice.
• Other important Vedic rituals:
o Asvamedha: Horse sacrifice meant to establish a king’s supremacy over other kings.
o Semontonnayam: A ceremony to ensure the safety of the child in the womb.
o Jatkarma: A birth ceremony performed before the cutting of the umbilical cord.
o Upanayana: An initiation ceremony to confer status to boys of the higher varnas in their eighth year.

Q 7.B
• A virus is a small parasite that cannot reproduce by itself. Once it infects a susceptible cell, however, a
virus can direct the cell machinery to produce more viruses. Most viruses have either RNA or DNA as
their genetic material (but no virus can have both). The nucleic acid may be single- or double-

3 www.visionias.in ©Vision IAS


stranded. The entire infectious virus particle, called a virion, consists of the nucleic acid and an outer shell
of protein. The simplest viruses contain only enough RNA or DNA to encode four proteins. The most
complex can encode 100 – 200 proteins. Hence statement 1 is correct.
• Viruses can cause both Viral hemorrhagic fevers (VHFs) and cancers. When viruses cause an infection,
they spread their DNA, affecting healthy cells' genetic makeup and potentially causing them to turn into
cancer. HPV (Human Papillomavirus) infections, for instance, cause the virus' DNA to combine with the
host's DNA, disrupting the normal function of cells. Hence statement 2 is correct.
• Antivirals are a class of medications that are used to treat viral infections. Most viral infections
resolve spontaneously in immunocompetent (having a normal immune response) individuals. The aim of
antiviral therapy is to minimize symptoms and infectivity as well as to shorten the duration of illness.
These drugs act by arresting the viral replication (or multiplication) cycle at various stages. Currently,
antiviral therapy is available only for a limited number of infections. Most of the antiviral drugs currently
available are used to treat infections caused by HIV, herpes viruses, hepatitis B and C viruses, and
influenza A and B viruses. Because viruses are obligate, intracellular parasites, it is difficult to find drug
targets that interfere with viral replication without also harming the host cells. Unlike other
antimicrobials (antibiotics), antiviral drugs do not deactivate or kill the microbe (in this case, the
virus) but act by inhibiting replication. Hence statement 3 is not correct.

Q 8.A
• Surendranath Banerjea was 1869-batch ICS officer but he was dismissed from the service, on the
flimsy ground in 1874. After that, he came to the arena of public life and began to participate in ‘public
life’ in Kolkata.
• In 1876, he along with Ananda Mohan Bose founded a political association ‘Indian Association’ in
Kolkata. The Indian Association was the most important of pre-Congress associations and aimed to
promote by every legitimate means the political, intellectual and material advancement of the people.
• During 1905-11 he took a leading part in the Bengal anti-partition movement but opposed boycott and
acts of violence. He was a champion of local self-government and founder of Ripon College (now
Surendranath College).
• He became a member of the Imperial Legislative Assembly in 1921 and was knighted the same year.

Q 9.B
• In pursuance of the announcement made in the Union Budget 2014-15, RBI in 2014 has issued draft
guidelines for setting up Small Finance Banks in the private sector. The small finance bank shall be
registered as a public limited company under the Companies Act, 2013. Small Finance Banks will be
licensed under Section 22 of the Banking Regulation Act, 1949 and governed by the provisions of the
Banking Regulation Act, 1949; Reserve Bank of India Act, 1934, etc. Hence, statement 1 is correct.
• The objectives of setting up of small finance banks will be for furthering financial inclusion by
o Provision of savings vehicles primarily to unserved and underserved sections of the population
o Supply of credit to small business units; small and marginal farmers; micro and small industries; and
other unorganized sector entities, through high technology-low cost operations
• SFBs can be set up by Resident individuals/professionals, singly or jointly, each having at least 10 years
of experience in banking and finance at a senior level; and Companies and Societies in the private sector,
that are owned and controlled by residents, and having a successful track record of running their
businesses for at least a period of five years.
• Small finance banks will have to subject to the condition that the requirement of opening at least 25
percent of its banking outlets in unbanked rural centers (population up to 9,999 as per the latest
census). Hence, statement 2 is correct.
• The small finance banks shall primarily undertake basic banking activities of acceptance of deposits and
lending to unserved and underserved sections including small business units, small and marginal farmers,
micro and small industries and unorganized sector entities. SFBs can also undertake other non-risk
sharing simple financial services activities such as the distribution of mutual fund units, insurance
products, pension products, etc. with the prior approval of the RBI and after complying with the
requirements of the sectoral regulator for such products.
• The small finance bank will be subject to all prudential norms and regulations of RBI as applicable to
existing commercial banks including the requirement of maintenance of Cash Reserve Ratio (CRR) and
Statutory Liquidity Ratio (SLR). In view of the objectives for which small finance banks are set up,
the bank will be required to extend 75 percent of its Adjusted Net Bank Credit (ANBC) to the
sectors eligible for classification as priority sector lending (PSL) by RBI.

4 www.visionias.in ©Vision IAS


• Recently RBI allowed Payments Banks to apply for conversion into SFBs after five years of operations
if they are otherwise eligible as per these guidelines. Hence, statement 3 is not correct.

Q 10.C
• Statement 1 is not correct: Sikkim and Meghalaya are the only two states in India which are bordered by
just one Indian state each. Meghalaya is surrounded by Assam to its north and Bangladesh to its south.
The fact remains that a major part of the border of the state is situated on the north and eastern part of
Assam. The south and west part are with Bangladesh. Sikkim is bordered by the Tibet Autonomous
Region of China to the north and northeast, by Bhutan to the southeast, by the Indian state of West
Bengal to the south, and by Nepal to the west. Thus Sikkim is bordered by just one Indian state which is
West Bengal while Meghalaya is bordered only by Assam. Goa, on the other hand, is bordered by both
Maharashtra and Karnataka.
• Statement 2 is correct: 82° 30′ East is the Indian Standard Meridian, it passes through Mirzapur in Uttar
Pradesh and touches five states of India. The states it passes through are Uttar Pradesh, Madhya Pradesh,
Chhattisgarh, Orissa, and Andhra Pradesh. Tropic of Cancer passes through Mizoram, Tripura, West
Bengal, Jharkhand, Chhattisgarh, Madhya Pradesh, Rajasthan, and Gujarat. Thus Chhatisgarh and
Madhya Pradesh are the only two states through which both the lines pass.
• Statement 3 is correct: Jammu and Kashmir is not a state anymore. West Bengal also shares its borders
with three countries because of its narrow northward extension. Those countries are-Bangladesh to the
East and Nepal and Bhutan in the North. Punjab Rajasthan and Gujarat share borders only with Pakistan.
Assam shares international borders with Bangladesh and the Kingdom of Bhutan. Arunachal Pradesh
shares international borders with Bhutan in the west, Myanmar in the east, and a disputed border with
China in the north at the McMahon Line.

Q 11.C
• The Asur tribe has been using mobile radio to transmit local news and songs which is helping them
revive the dying tribal language.
• About Asur Tribe
o Asur Tribe is an Austroasiatic ethnic group living primarily in the Indian state of Jharkhand. They
are included in the list of Particularly Vulnerable Tribal Groups (PVTG’s).
o The main festival of the tribe includes Sarhul, Phagua, Navakhans among others.
o As per the 2011 census, the tribe has a population of around 23,000 in Latehar and Gumla districts.
o In the community, 50% of the population could barely speak in Asur language; they are not fluent in
the language.
o The Asur language figures in the list of UNESCO Interactive Atlas of the World’s Languages in
Danger.
o Only 7,000 to 8,000 Asur tribals are left in the community who are well conversant in the language.

Q 12.A
• The number of animals in a population is conventionally estimated by capture-recapture without modeling
the spatial relationships between animals and detectors. Problems arise with non‐spatial estimators when
individuals differ in their exposure to traps or the target population is poorly defined. Spatially explicit
capture-recapture (SECR) methods devised recently to estimate population density largely avoid these
problems. Spatially explicit capture-recapture (SECR) models have gained enormous popularity to solve
abundance estimation problems in ecology.
• Extract compare is Conservation Research Ltd's software designed to automatically identify individual
animals from their natural markings. It uses still photos were taken by researchers, tourists or camera
traps and is applicable to any species with natural markings.
• In India, tiger faeces were used for genetic sampling and estimating the big cats' population for the 2018
Tiger Census. The experiment was called ‘scats’ and it was conducted for the first time in uneven
terrains. The camera traps were also used in tiger estimation.
• In addition to the above M-STrIPES (Monitoring System For Tigers-Intensive Protection and
Ecological Status) is the other application software used for tiger estimation.

Q 13.B
• Statement 1 is not correct: CGA works under the Ministry of Finance, Government of India. Though it
itself is not a constitutional body, it derives its mandate from Article 150 of the Constitution which says:
"The accounts of the Union and of the States shall be kept in such form as the President may, on the

5 www.visionias.in ©Vision IAS


advice of the Comptroller and Auditor-General of India, prescribe". Thus while CAG (Comptroller
and Auditor General of India) is an independent constitutional body, CGA works under the Ministry of
Finance and not a constitutional body.
• Statement 2 is correct: It is the principal account adviser or the apex accounting authority of the Union
Government. Its statutory mandate (duties and functions) have been defined in the Allocation of Business
Rules, 1961
• Matters relating to the Controller General of Accounts include-
o general principles of Government accounting relating to Union or State Governments and form of
accounts, and framing or revision of rules and manuals relating thereto;
o reconciliation of cash balance of Union Government with Reserve Bank in general and, in particular,
of Reserve Deposits pertaining to Civil Ministries or Departments;
o overseeing the maintenance of adequate standards of accounting by Central Civil Accounts Offices;
o consolidation of monthly accounts, preparation of the review of trends of revenue realization and
significant features of expenditure, etc. and preparation of annual accounts (including Summary, Civil
Appropriation Accounts) showing under the respective heads, the annual receipts and disbursements
for the purpose of the Union Government;
o administration of Central Treasury Rules and Central Government Account (Receipts and Payments
Rules 1983);
o coordination and assistance in the Introduction of management accounting system in Civil Ministries
or Departments;(g) cadre management of Group 'A' (Indian Civil Accounts Service) and Group 'B'
Officers of the Central Civil Accounts Offices;
o matters relating to the Central Civil Accounts staff belonging to Group ‘C’ and ‘D’;
o disbursement of Pension through Public Sector Banks (PSBs) in respect of Central Civil Pensioners,
Freedom Fighters, High Court Judges, Ex-M.P.s and Ex-Presidents.
• Statement 3 is not correct: The CAG is also the head of the Indian Audit and Accounts Department, the
affairs of which are managed by officers of Indian Audit and Accounts Service.

Q 14.D
• In the healthcare setting, “alcohol” refers to two water-soluble chemical compounds—ethyl alcohol and
isopropyl alcohol—that have generally underrated germicidal characteristics. These alcohols are
rapidly bactericidal (bacteria-killing) rather than bacteriostatic (which stops bacteria from
reproducing or multiplying) against vegetative forms of bacteria; they also are tuberculocidal,
fungicidal, and virucidal but do not destroy bacterial spores. Their cidal activity drops sharply when
diluted below 50% concentration, and the optimum bactericidal concentration is 60%–90% solutions in
water (volume/volume). Methyl alcohol (methanol) has the weakest bactericidal action of the alcohol and
thus seldom is used in healthcare. Also, methanol is highly toxic, and severe systemic toxicity and even
deaths can occur after oral, pulmonary and/or skin exposures. Hence statement 2 is not correct.
• The most feasible explanation for the antimicrobial action of alcohol (or alcohol-based sanitizers) is
the denaturation of proteins. This mechanism is supported by the observation that absolute ethyl
alcohol, a dehydrating agent, is less bactericidal than mixtures of alcohol and water because proteins are
denatured more quickly in the presence of water. In simple words, Ethyl alcohol (also called ethanol) kills
bacteria by dissociating/dissolving (and not vaporize) the bacteria's cellular membrane. The way ethanol
dissolves a cell membrane is by having one end that dissolves well in water (the one with the OH) and the
other end that dissolves well in fatty/greasy like substances (the end with the CH2CH3). Bacteria cell
membranes are like water on the outside and like fat on the inside, so when you apply ethanol to a cell
membrane, instead of all the parts of the membrane sticking together, they'll associate with the ethanol
causing the membrane to fall apart. Hence statement 1 is not correct.

6 www.visionias.in ©Vision IAS


Q 15.C

• Additional Information:
o Shaivism emerged as a ‘state’ cult under Rajaraja I (A.D 985-1045.), Rajendra I and Kulottunga II
(A.D.1133-50.). The large-scale construction of the Shaiva temples, especially in the royal capital of
Gangaichondacholapuram and Tanjavur projected the Chola policy of promoting Shaiva bhakti. The
collection of the Shaiva hymns and the composition of the hagiographies were a part of the royal
project that contributed to the evolution of a Shaiva scripture (marai). Nambi Andar Nambi, the
compiler of the Thirumurai (the Shaiva scripture) and Cekkilar, the composer of the Shaiva
hagiography, the Periya Puranam, were associated with the court of Rajaraja I and Kulottunga II
respectively.
o About Thirumai: It is a twelve volume compendium of songs or hymns in praise of Shiva in the
Tamil language from the 6th to the 11th century by various poets in South India.
✓ Nambi Andar Nambi compiled the first seven volumes by Appar, Sampandhar, and
Sundarar as Tevaram during the 12th century.
✓ Tiruvacakam and Tirukovayar by Manickavasagar are included as the eighth, nine parts
are compiled as the ninth Tirumurai out of which most are unknown.
✓ Tenth as Tirumandiram by Tirumular, the famous Siddhar.
✓ The eleventh is compiled by Karaikal Ammaiyar, Cheraman Perumal and others.

Q 16.B
• When the government is going to the financial market to raise money, it can do it by issuing two types of
debt instruments – treasury bills and government bonds. Treasury bills are issued when the government
needs money for a shorter period while bonds are issued when it need debt for more than say five years.
• Treasury bills generally shortened as T-bills, have a maximum maturity of 364 days. Hence, they are
categorized as money market instruments (money market deals with funds with a maturity of less than
one year).
• Zero-coupon security is a debt security that does not pay interest but instead trades at a deep
discount, rendering a profit at maturity, when the bond is redeemed for its full face value. Treasury bills
are regarded as zero-coupon securities as they pay no interest. Rather, they are issued at a discount (at
a reduced amount) and redeemed (given back money) at the face value at maturity. Hence option (b) is
the correct answer.
• Treasury bills are presently issued in three maturities, namely, 91 days, 182 days and 364 days.
• The return to the investors is the difference between the maturity value or the face value and the issue
price.
• The regular Treasury bills have no inflation protection but Treasury Inflation-Protected Securities
(TIPS) do.

Q 17.B
• The Belgaum border dispute is a dispute involving the Indian states of Karnataka and
Maharashtra. Belgaum is a part of Karnataka and was earlier part of the British India's Bombay
Presidency. The Bombay Presidency encompassed present-day Gujarat, Maharashtra as well as certain
areas of North Karnataka. Hence pair 1 is not correctly matched.
• Assam and Meghalaya are two states embroiled in a border dispute for decades now. It first started when
Meghalaya challenged the Assam Reorganisation Act of 1971, which gave part of the Mikir Hills to
7 www.visionias.in ©Vision IAS
Assam, which according to Meghalaya, are part of United Khasi and Jaintia Hills. Langpih - or Lumpi,
as it called in Assam - has been a bone of contention between the two north-eastern neighbours for
decades. Meghalaya claims it to be part of West Khasi Hills district, while Assam asserts that it falls
under its Kamrup district. Hence pair 2 is correctly matched.
• The dispute between Gujarat and Rajasthan relates to Mangadh Hill, located on the border of the
two states. Gujarat claims half of the hill, while Rajasthan claims the entire hill. The dispute is 40-
year-old, though the Rajasthan Government presently has control over it. The Panchmahal district
administration of Gujarat recently started constructing a road to reach the hill, along with forest huts and
hand pumps for the pilgrims. The Rajasthan government has raised strong objections to this. Hence pair 3
is correctly matched.

Q 18.A
A Few Important Features of the Act are:
• It ended the British rule in India and declared India as an independent and sovereign state from August 15,
1947.
• It provided for the partition of India and the creation of two independent Dominions of India and Pakistan
with the right to secede from the British Commonwealth.
• It deprived the British Monarch of his right to veto bills or ask for a reservation of certain bills for his
approval. But, this right was reserved for the Governor-General. The Governor-General would have full
power to assent to any bill in the name of His Majesty. Hence statement 1 is correct.
• It designated the Governor-General of India and the provincial governors as constitutional (nominal)
heads of the states. They were made to act on the advice of the respective council of ministers in all
matters.
• It dropped the title of Emperor of India from the royal titles of the king of England.
• It empowered the Constituent Assemblies of the two dominions to frame and adopt any constitution for
their respective nations and to repeal any act of the British Parliament, including the Independence
act itself.
• It empowered the Constituent Assemblies of both the dominions to legislate for their respective territories
till the new constitutions were drafted and enforced. No Act of the British Parliament passed after August
15, 1947, was to extend to either of the new dominions unless it was extended thereto by a law of the
legislature of the dominion.
• It provided for the governance of each of the dominions and the provinces by the Government of India
Act of 1935, till the new Constitutions were framed. The dominions were however authorized to make
modifications in the Act. Hence statement 2 is not correct.

Q 19.A
• A convict who is under the sentence of death is allowed to file a mercy petition within a specific period of
seven days after the date on which the Superintendent of Jail informs him about the rejection of the appeal
or special leave to appeal by the Supreme Court.
• The process begins with filing a mercy petition with the President under Article 72 of the
Constitution. The petition is then sent to the Ministry of Home Affairs in the Central Government
for inspection. The petition is then discussed by the Home Ministry in deliberation with the concerned
State Government. After the deliberation, a suggestion is made by the Home Minister and then the petition
is again sent to the President. The advice and suggestions bind the President. Hence statement 3 is not
correct.
• The mercy petition can be made to both president and the governor. Hence statement 2 is correct.
• The extent of the pardoning power of the President under Article 72 is broader than the pardoning power
of the Governor under Article 161. The power varies in the following two ways:
o The power of the President to grant pardon expands in the cases where the punishment or sentence is
by a Court Martial but Article 161 does not impart any such power to the Governor.
o The President can grant pardon in all the cases where the sentence is given is a sentence of death but
the pardoning power of Governor does not stretch to death sentence cases. However, the Governor
can commute the death sentence to life imprisonment (In the case of Rajiv Gandhi assassination,
the Governor of Tamil Nadu had commuted the death sentence of a convict to life
imprisonment).
• The mercy petition is however upon the discretion of the president and governor and the convict cannot
claim it as a right. The case of Ranga Billa in the Supreme Court was called upon to decide the nature and

8 www.visionias.in ©Vision IAS


ambit of the pardoning power of the President of India under Article 72 of the Constitution. In this case,
the death sentence of one of the appellants was confirmed by the Supreme Court and mercy petition was
also rejected by the President. Then, the appellant filed a writ petition in the Supreme Court challenging
the discretion of the President to grant pardon on the ground that no reasons were provided for rejection of
his mercy petition. The court rejected the petition and stated that the term “pardon” itself signifies that it is
an entirely discretionary remedy and grant and the rejection of it need not be reasoned. Hence statement
1 is correct.

Q 20.D
• The United Nations Conference on Trade and Development (UNCTAD), United Nations Department
of Economic and Social Affairs (UN DESA) and the five United Nations Regional Economic
Commissions have jointly released the World Economic Situation and Prospects 2020 (WESP)
recently. Hence option (d) is correct.
• Other Reports published by UNCTAD:
o Trade and Development Report
o World Investment Report
o The Least Developed Countries Report
o Information and Economy Report
o Technology and Innovation Report
o Commodities and Development Report

Q 21.B
• PPIs are instruments that facilitate the purchase of goods and services, including financial services,
remittance facilities, etc., against the value stored on such instruments. The value stored on such
instruments represents the value paid for by the holders by cash, by debit to a bank account, or by credit
card. Hence, statement 1 is correct.
• The pre-paid instruments can be issued as smart cards, magnetic stripe cards, internet accounts, internet
wallets, mobile accounts, mobile wallets, paper vouchers and any such instrument which can be used to
access the pre-paid amount.
• A company incorporated in India and registered under the Companies Act, 1956 / Companies Act, 2013,
having a minimum paid-up capital of Rs. 5 crore and minimum positive net worth of Rs. 1 crore at all the
times are permitted to issue PPIs in India.
• PPIs can be reloadable or non-reloadable. The loading/reloading of PPIs shall be through payment
instruments issued by entities regulated in India and shall be in Indian Rupees (INR) only. Banks
are permitted to issue and reload such payment instruments at their branches and ATMs against payment
by cash/debit to bank account/credit card and through their business correspondents (BCs). In the case of
non-reloadable PPIs, the outstanding amount in it can be transferred to a new similar PPI of the same
issuer, upon expiry. Hence, statement 2 is correct.
• PPIs that can be issued in the country are classified under three types viz. (i) Closed System PPIs, (ii)
Semi-closed System PPIs, and (iii) Open System PPIs.
o Closed System PPIs: These are payment instruments issued by an entity for facilitating the purchase
of goods and services from it. These instruments do not permit cash withdrawal or redemption. As
these instruments do not facilitate payments and settlement for third party services, the issue and
operation of such instruments are not classified as payment systems. Hence, RBI approval is not
required for issuing them. Eg. Many of the web portals for online purchases /shopping - Make my
Trip, Flipkart, Jabong, etc. run wallets for its customers. Pre-paid cards of mobile companies also
belong to this category.
o Semi-closed System PPIs: These are payment instruments that can be used for the purchase of goods
and services, including financial services at a group of clearly identified merchant locations/
establishments which have a specific contract with the issuer to accept the payment instruments.
These instruments do not permit cash withdrawal or redemption by the holder. Such PPIs are non-
reloadable in nature; The above two can be issued only in electronic form.
o Open System PPIs: These PPIs are issued only by banks (approved by RBI) and are used at any
merchant for purchase of goods and services, including financial services, remittance facilities, etc.
Cash withdrawal at ATMs / Points of Sale (PoS) terminals / Business Correspondents (BCs) are also
allowed through such PPIs.
• No interest is payable on PPI balances. Hence, statement 3 is not correct.

9 www.visionias.in ©Vision IAS


Q 22.D
• Ecological efficiency is the amount of energy that is transferred from one trophic level to the
next. It is defined as the energy supply available to trophic level N + 1, divided by the energy consumed
by trophic level N. This follows the 10% rule, which states that roughly 10% of the energy at one level
will be available to be used by the next level. Hence if ecosystem A has higher ecological efficiency than
B, then it means energy transfer between trophic levels is more efficient in ecosystem A than B. Hence
option (d) is the correct answer.
• For example, in an ecological pyramid, producers are consumed by primary consumers. The consumers
eat these producers, but only about 10% of what they consume actually becomes new biomass for the
primary consumer. So if a wolf eats a deer, only 10% of what was consumed becomes new wolf biomass.
Q 23.C
• Statement 3 is correct: Macaulay’s Minutes' formed the basis for the English Education
Act of 1835. Lord Macaulay held the view that “Indian learning was inferior to European
learning”.
• Hence, Macaulay's ‘Minute on Indian Education’ proposed that the limited government
resources were to be devoted to the teaching of Western sciences and literature through the
medium of the English language alone.
• In his Minute he also advocated that the government try to educate only a few Indians. Thus
creating a class “Indian in blood and color but English in tastes, in opinions, in morals and in
intellect” who would, in turn, teach the rest of the masses. This is called the ‘downward
filtration’ policy.
• Statement 1 is not correct: Macaulay’s proposals were promptly accepted by the government
and soon made English as the medium of instruction in its schools and colleges and opened a few
English schools and colleges instead of a large number of elementary schools, thus neglecting
mass education.
• The direction and the philosophy set by the Minutes were executed by the Act.
• Wood’s Despatch (1854) asked the government of India to assume responsibility for the
education of the masses.
• Statement 2 is not correct: The Act did not provide the fund for the Orientalist institution
for their continuation. Instead, it stopped the promotion of Oriental institutions like Calcutta
Madrasa and Benaras Sanskrit College.
Q 24.A
• The Brahmaputra, one of the largest rivers of the world, has its origin in the Chemayungdung glacier of
the Kailash range near the Mansarovar lake. The Brahmaputra, a trans-boundary river, is unstable in its
entire reach in Assam except for a few places and well-known for floods, channel shifting and bank
erosion. This is due to the fact that most of its tributaries are large, and bring large quantity of sediments
owing to heavy rainfall in its catchment area. Hence, statement 1 is not correct and statement 2 is
correct.
• Additionally, the entire area falls in an earthquake-prone zone and experiences high rainfall. Landslides
and earthquakes send in a lot of debris in the rivers, causing the river bed to rise and leading to
floods. Hence, statement 3 is correct.
Q 25.C
• Recently Prime Minister of India visited Kingdom of Saudi Arabia and attended Future Investment
Initiative (FII) held in Riyadh.
• Major outcomes of the visit:
o Establishment of a Strategic Partnership Council (SPC). India has become the fourth country to
sign such an agreement with Saudi Arabia. The SPC will have two parallel tracks:
▪ Political, security, culture and society, headed by Foreign Ministers of both the countries;
▪ Economy and investment, headed by India’s Commerce Minister and Saudi’s Energy Minister.
• The joint statement rejected all forms of interference in the internal affairs of countries. This is seen as
Saudi Arabia’s tacit support for India’s decision to revoke the special status of Jammu and Kashmir.
• Both countries signed 12 MoUs on issues such as preventing narcotics trafficking, renewable energy,
security collaboration etc.
• Saudi Arabia hosts a 2.6 million Indian community that sends home remittances of $11 billion or more
annually.
• Hence option (c) is the correct answer.
10 www.visionias.in ©Vision IAS
Q 26.B
• The Reserve Bank of India (established in the year 1935 in accordance with the provisions of the Reserve
Bank of India Act, 1934), is not just the monetary authority but also the highest supervisor and regulator
of the banking system. It prescribes broad parameters of banking operations within which the country's
banking and financial system functions such as issuing licenses, branch expansion, liquidity of assets, the
amalgamation of banks, etc. In the given context, the seventy-year-old man uses two bank accounts for
carrying out transactions, irrespective of the nature of the banks (private and public) they come under the
purview of the Reserve Bank of India. Hence option 1 is correct.
• The Securities and Exchange Board of India (SEBI) is a statutory body established on April 12, 1992, in
accordance with the provisions of the Securities and Exchange Board of India Act, 1992. The basic
functions of the Securities and Exchange Board of India are to protect the interests of investors in
securities and to promote and regulate the securities market. In the given context, the seventy-year-old
man is trading on the intraday platform (which involves speculation in securities, specifically buying and
selling financial instruments within the same trading day) which comes under the purview of the
SEBI. Hence option 2 is correct.
• The Telecom Regulatory Authority of India (TRAI) has come into being as a consequence of the opening
up of the telecommunication sector to private operators. It was set up under the TRAI Act of 1997. It has
the power
o to recommend the need for and timing of introduction of new service providers and terms and
conditions of the license to a service provider;
o and to protect consumers’ interest, monitor quality of services, inspect equipment used in networks
and make recommendations about such equipment;
• The seventy-year-old man uses multiple apps to avail services e.g. a trading app, BHIM app, etc which
further use mobile data or internet services (telecom services). Thus the mobile/internet service provider
comes within the ambits of TRAI. Hence option 3 is correct.
• Insurance Regulatory and Development Authority of India or the IRDAI is the apex body responsible for
regulating and developing the insurance industry in India. It is an autonomous body. Even though the
individual in the given context might have availed insurance schemes, nothing as such is mentioned in the
question statement. Hence option 4 is not correct.
• The interim PFRDA was established in 2003. This was to oversee the National Pension System (NPS) and
regulate India’s pensions sector. The interim PFRDA transitioned into the PFRDA with the passage of the
Pension Fund Regulatory Development Authority (PFRDA) Act, 2013. In the given context, there is no
mention of the individual using his pension funds. Hence option 5 is not correct.

Q 27.D
• The Flame-throated Bulbul is a member of the bulbul family of passerine birds. It is endemic to the
Western Ghats, where it can often be found in scrubby foothill forest, as well as overgrown and degraded
forest edge.
• The flame-throated bulbul, also called the Rubigula, was chosen as the mascot of the 36th National Games
to be held in Goa, because it is the State bird of Goa. Its IUCN status is ‘Least Concern’.

Q 28.A
• The Genetic Engineering Appraisal Committee (GEAC) functions in the Ministry of Environment,
Forest and Climate Change (MoEF&CC). ItThis document
is responsible for is
thepersonalised for Shubham
appraisal of activities involvingTanwar(tan
large
scale use of hazardous microorganisms and recombinants in research and industrial production from the
environmental angle. The committee is also responsible for the appraisal of proposals relating to the
release of genetically engineered (GE) organisms and products into the environment including
experimental field trials. GEAC is chaired by the Special Secretary/Additional Secretary of MoEF&CC
and co-chaired by a representative from the Department of Biotechnology (DBT). Hence statement 1 is
correct.
• There is a well established regulatory framework for approval of GM Crops as per “Rules for the
Manufacture/Use/Import/Export and Storage of Hazardous Microorganisms, Genetically
Engineered Organisms or Cells, 1989” under the Environment (Protection) Act, 1986 in the
Country. Evaluation of each application of GM crop is done on a case-to-case basis after a thorough
examination of health, environment, food and feed safety assessment studies undertaken in a systematic
and scientific manner as per prescribed guidelines, manuals and standard operating procedures stipulated
by various regulatory agencies under the Rules, 1989 from time to time. Hence statement 2 is not
correct.

11 www.visionias.in ©Vision IAS


Q 29.A
• India enjoys an important position in the global pharmaceuticals sector. It is the largest provider of
generic medicines globally, occupying a 20% share in global supply by volume, and also supplies 50% of
global demand for vaccines the largest in the world.
• India ranks 3rd worldwide for production by volume and 13th by value, thereby accounting for
around 10% of the world’s production by volume and 1.5% by value. Hence statement 1 is not correct.
• The pharmaceutical industry was valued at $ 36.7 bn in 2018. The market is expected to expand at a
CAGR of 22.4% over 2015–20 to reach $ 55 billion. India’s pharmaceutical exports stood at US$ 17.27
billion in FY18 and have reached US$ 19.14 billion in FY19. Pharmaceutical exports include bulk drugs,
intermediates, drug formulations, biologicals, Ayush & herbal products and surgical.
• Generic drugs, with 71% market share, form the largest segment of the Pharmaceutical industry in
India. This is set to grow as exports of generics to the US rise, as branded drugs worth US$ 55 billion will
become off-patent during 2017-2019. Hence statement 2 is not correct.
• India's biotechnology industry comprising bio-pharmaceuticals, bio-services, bio-agriculture, bio-industry,
and bioinformatics is expected to grow at an average growth rate of around 30 percent a year and reach
US$ 100 billion by 2025.
• Some of the initiatives taken by the government to promote the pharmaceutical sector in India are as
follows:
o The Drug Controller General of India (DCGI) announced its plans to start a single-window facility to
provide consents, approvals and other information. The move is aimed at giving a push to the Make in
India initiative.
o The Government of India is planning to set up an electronic platform to regulate online pharmacies
under a new policy, in order to stop any misuse due to easy availability.
o The Government of India unveiled 'Pharma Vision 2020' aimed at making India a global leader in
end-to-end drug manufacture. Approval time for new facilities has been reduced to boost investments.
o The government introduced mechanisms such as the Drug Price Control Order and the National
Pharmaceutical Pricing Authority to deal with the issue of affordability and availability of medicines.
o 100% Foreign Direct Investment (FDI) is allowed under the automatic route for greenfield pharma,
whereas in brownfield pharma 74% is allowed under the automatic route and thereafter through
government approval route. Hence statement 3 is correct.
Q 30.B
• Operation Twist is a move in which a central bank decides to simultaneously buy long-dated securities
while selling short-term securities. This is the first time RBI has undertaken such an unconventional
policy measure with the aim of flattening the yield curve by lowering longer-term rates to boost lending
and growth.
• Operation Twist normally leads to lower longer-term yields, which will help boost the economy by
making loans less expensive for those looking to buy homes, cars and finance projects, while saving
becomes less desirable because it doesn’t pay as much interest. Hence statement 1 is not correct.
• As the central bank buys more long-term security and sells off short term bonds, the bond yield i.e. the
return an investor gets on his holding - comes down significantly. Since long-term bond yield (10-year
government securities) is a key market interest rate, lower rates can help people avail more long-term
loans. It also helps in bringing down overall borrowing costs for the government. Hence, statement 2 is
correct.
• Lower long-term interest rates would provide more liquidity to the market. Anything that increases the
demand for long-term government bonds puts downward pressure on the interest rate and less demand for
bonds tends to put upward pressure on interest rates. Hence, statement 3 is correct.
Q 31.B
• Recently, India celebrated the 640th birth anniversary of Saint Ravidas. Ravidas was an Indian
mystic poet-saint of the Bhakti movement during the 15th to 16th century CE.
• He is revered as "Guru Ravidas", in most of the North India and Deccan Region. He was born in and
around 1450 C.E. during Lodhi Dynasty & died around 1520 C.E. His poems are included in Guru Granth
Sahib. He is considered as a disciple of Ramananda. He is contemporary to Kabir, Meera, Guru Nanak
Dev Ji.
• Ravidas taught removal of social divisions of caste and gender, and promoted unity in the pursuit of
personal spiritual freedoms. He followed "nirguna" form of supreme being and discussed the comparisons
with "saguna". He worshippped god as "Hari".
• His theory propagated around a place of "Begumpura", which is a place where there is no sorrow.
The place where Ravidas is worshipped is modeled around this theory of "Begumpura".

12 www.visionias.in ©Vision IAS


Q 32.C
• Article 118(1) of the Constitution empowers each House of Parliament to make rules for regulating its
Procedure and the Conduct of its business. Under this provision of the Constitution, Rajya Sabha and Lok
Sabha have adopted rules for regulating their procedure and the conduct of their business.
• Under Rule 69 of Rajya Sabha and Rule 74 of the Lok Sabha, When a Bill is introduced or on
some subsequent occasion, the member in charge may make one of the following motions in regard to
one’s own Bill, namely:
o that it be taken into consideration; or
o that it be referred to a Select Committee of the House; or
o that it be referred to a Joint Committee of the Houses with the concurrence of the Council; or
o that it be circulated for the purpose of eliciting opinion thereon: Provided further that no such
motion shall be madeuntil after copies of the Bill have been made available for the use of
members, and that any member may object to any such motion being made unless copies of the
Bill have been so made available for two days before the day on which the motion is made and
such objection shall prevail, unless the Speaker allows the motion to be made.
• Recently, the Constitutional (One Hundred and Third) Amendment Act 2019 providing reservation for
Economically Weaker Sections was brought for consideration of Parliament in less than 48 hours from the
time the decision was taken by the Centre. By doing so, the government ensured that there was
insufficient time for Parliament scrutiny. The Bills around J&K also suffered from a similar defect and
was allowed to be introduced by the presiding officer of the house.
• A Bill undergoes three readings in each House, i.e., the Lok Sabha and the Rajya Sabha before it is
submitted to the President for assent.
o The First Reading refers to (i) motion for leave to introduce a Bill in the House on the adoption of
which the Bill is introduced; or (ii) in the case of a Bill originated in and passed by the other House,
the laying on the Table of the House of the Bill, as passed by the other House.
o The Second Reading consists of two stages. The "First Stage" constitutes discussion on the principles
of the Bill and its provisions generally on any of the following motions - that the Bill be taken into
consideration; or that the Bill be referred to a Select Committee of the House; or that the Bill be
referred to a Joint Committee of the Houses with the concurrence of the other House; or that the Bill
be circulated for the purpose of eliciting opinion thereon. The "Second Stage" constitutes the clause
by clause consideration of the Bill, as introduced in the House or as reported by a Select or Joint
Committee, as the case may be. In the case of a Bill passed by Rajya Sabha and transmitted to Lok
Sabha, it is first laid on the Table of Lok Sabha by the Secretary-General, Lok Sabha. In this case, the
Second Reading refers to the motion (i) that the Bill, as passed by Rajya Sabha, be taken into
consideration; or (ii) that the Bill be referred to a Select Committee (if the Bill has not already been
referred to a Joint Committee of the Houses).
o The Third Reading refers to the discussion on the motion that the Bill or the Bill, as amended,
be passed. Hence option(c) is not correct as a bill cannot be put to vote immediately after its
introduction.
Q 33.A
• Genome editing can be performed in germline cells (sperm, eggs or embryos) to induce heritable genetic
changes or in somatic cells, also called non-reproductive cells (other cells) to induce non-heritable
changes. Somatic cell editing is much closer to clinical implementation, yet is lesser-known than germline
editing. The first type of genome editing is called germline editing while the second type is called Somatic
Cell Genome Editing.
• Somatic cells are any group of cells of a living organism other than the reproductive cells which can be
nerve cells (neurons), skin cells, or blood cells (Red Blood Cells and White Blood Cells). Even testicular
cells like Sertoli, Leydig, and peritubular cells are somatic cells that interact with germ cells (egg or ovum
and sperm) during spermatogenesis. Hence only options 2, 3 and 4 are correct.

Q 34.B
• Based on the disease prevalence, incidence, and the known or unknown disease pathways, there are
several ways an epidemiologist might describe a disease event:
o Sporadic refers to a disease that occurs infrequently or irregularly. Foodborne pathogens, such as
Salmonella or E. coli, can often cause sporadic disease outbreaks.
o Cluster refers to a disease that occurs in larger numbers even though the actual number or cause may
be uncertain. An example is the cluster of cancer cases often reported after a chemical or nuclear plant
disaster.

13 www.visionias.in ©Vision IAS


o Endemic refers to the constant presence and/or usual prevalence of a disease in a geographic
population.
o Hyperendemic refers to persistent, high levels of disease well above what is seen in other populations.
For example, HIV is hyperendemic in parts of Africa, whereas many as one in five adults has the
disease, and endemic in the United States, where roughly one in 300 is infected.6
o According to WHO Epidemic refers to the occurrence in a community or region of cases of an
illness, specific health-related behaviour, or other health-related events clearly in excess of
normal expectancy. The community or region and the period in which the cases occur are specified
precisely. The number of cases indicating the presence of an epidemic varies according to the agent,
size, and type of population exposed, previous experience or lack of exposure to the disease, and time
and place of occurrence. Hence pair 3 is not correctly matched.
o According to WHO, a disease outbreak is the occurrence of disease cases in excess of normal
expectancy. The number of cases varies according to the disease-causing agent, and the size and
type of previous and existing exposure to the agent. Hence pair 2 is correctly matched.
o According to WHO, pandemic refers to an epidemic that has spread over several countries or
continents, usually affecting a large number of people. Hence pair 1 is not correctly matched.

Q 35.C
• The Group of Seven (G7) is an informal bloc of industrialized democracies—Canada, France, Germany,
Italy, Japan (only Asian country), the United Kingdom, and the United States—that meets annually to
discuss issues such as global economic governance, international security, and energy policy. Hence
statement 2 is not correct.
• Russia belonged to the forum from 1998 through 2014, when the bloc was known as the Group of Eight
(G8), but was suspended following its annexation of Crimea.
• France, Italy, Japan, the United Kingdom, the United States, and West Germany formed the Group of Six
in 1975 (Canada joined the following year) to provide a platform for the non-communist powers to
address pressing economic concerns, which included inflation and a recession sparked by the
Organization of the Petroleum Exporting Countries (OPEC) oil embargo. Hence statement 1 is
correct.
• European Union has participated fully in the G7 since 1981 as a “non-enumerated” member. It is
represented by the presidents of the European Council, which comprises the EU member states’ leaders,
and the European Commission, the EU’s executive body. There is no formal criteria for membership, but
the participants are all developed democracies.
• FATF is the Financial Action Task Force, an inter-governmental body established in Paris in 1989
by the Group of 7 (G7). It seeks to combat money laundering, terrorist financing and other threats to the
international financial system. It is both a policy-making and enforcement body. Hence statement 3 is
correct.

Q 36.A
• The Union List (Schedule 7, List I) recognizes a distinction between taxes and fees. In the Union List
entries, 82 to 92A relate to taxes and duties while entry 96 specifies the legislative power for fees in
respect of any of the matters in the Union List, except the fees taken in any court.
• It is in view of the separate entries relating to taxes and fees in the Seventh Schedule that the Supreme
Court has held that our Constitution recognizes a ‘different and distinct connotation between taxes and
fees'.
• The concepts of tax and fee were examined by a Constitution Bench of the Supreme Court in considerable
detail for the first time in the case of Commissioner, Hindu Religious Endowments, Madras v. Sri
Lakshmindra Tirtha Swamiar of Sri Shirur Mutt.
• The Supreme Court accepted the definition of tax as “a compulsory exaction of money by public
authority for public purposes enforceable by law and is not payment for services rendered". The
Supreme Court concluded that the levy of a tax is for the purposes of augmenting general revenue
and there is no element of quid pro quo between the taxpayer and the public authority. Hence,
statement 1 is correct and 2 is not correct.
• Additionally, it was observed that as taxes are part of the common burden, the quantum of imposition
upon the taxpayer depends generally upon his or her capacity to pay.
• Further, all the tax monies collected by the Union Government are deposited in the Consolidated Fund of
India, subject to the exceptions carved out in Article 270. The tax monies can be spent towards any public
purpose as deemed fit by the Union Government.

14 www.visionias.in ©Vision IAS


• The Supreme Court in Commissioner, Hindu Religious Endowments, Madras v. Sri Lakshmindra Tirtha
Swamiar of Sri Shirur Mutt explained that the concept of fee is “a charge for a special service rendered
to individuals by some governmental agency". The Supreme Court held that the presence of quid pro
quo distinguishes a fee from a tax. However, the quid pro quo is not always a sine qua non for a fee. For
example, a regulatory fee is a license fee for which it is not necessary to establish the element of quid pro
quo (understood as a compensation for a service that has been provided).
• The quantum of the fee imposed should ideally be commensurate with the expenses incurred by the
Government in rendering the service. Additionally, a fee and a tax cannot be distinguished on the grounds
that the latter is a compulsory payment, while the former is not.

Q 37.A
• The Wildlife Protection Act, 1972, provides for protection to listed species of flora and fauna and
establishes a network of ecologically-important protected areas.
• The Wildlife Protection Act, 1972 empowers the central and state governments to declare any area
a wildlife sanctuary, national park or closed area. It provides for authorities to administer and
implement the Act; regulate the hunting of wild animals; protect specified plants, sanctuaries, national
parks, and closed areas; restrict trade or commerce in wild animals or animal articles; and miscellaneous
matters.
• The Act prohibits hunting of animals except with the permission of authorized officers when an animal
has become dangerous to human life or property or as disabled or diseased as to be beyond recovery. The
Act empowers the Central Government to include or exclude any species entry to or from any
schedule or transfer any species entry from one part of the schedule to another part of the same
schedule or from one schedule to another. Hence option (a) is the correct answer.

Q 38.B
• The temples in both Dravida and Nagara style are covered with elaborate sculpture and ornaments. The
placement of images in a temple was very carefully planned, for instance:
o River goddesses (Ganga and Yamuna) and mithunas (from the second century B.C. Indian art has
incorporated the symbol of loving couples, called mithuna, as most auspicious symbol for the health,
wealth and progeny, the three basic desires of human being as spelt in the Vedic literature) are usually
found at the entrance of a garbhagriha in a Nagara temple
o Dvarapalas (doorkeepers) are usually found on the gateways or gopurams of Dravida temples
o Images and sculptures of Nandi (bull carrier of Shiva) are commonly found in Dravida styles such as
in temples of Mahabalipuram and Kailashnath temple at Ellora.
o Hence only options 2 and 3 are correct.

Q 39.B
• Article 292 of the Indian Constitution states that the Government of India can borrow amounts
specified by the Parliament from time to time. Hence, statement 1 is correct.
• Public Finance (State) Division of Department of Expenditure (Ministry of Finance) deals with the
matters connected with state finances such as issues relating to release of Additional Central
Assistance/Special Assistance to the States for the projects/schemes for which budgetary allocation is
provided under the Demand operated by Department of Expenditure.
• Public Finance (States) Division also deals with Debt and liabilities management of States,
enforcement of the fiscal roadmap mandated by Finance Commissions through the powers to approve
borrowings by States under Article 293 (3) of the Constitution of India, Coordination with RBI to monitor
States’ debt, collating and maintaining state finance data, analysis of trends of State Finances, Scrutiny of
State Legislations having bearing on the State's Finances. Hence, statement 2 is not correct.
• The composition of State Government liabilities has changed significantly in the last 25 years. The share
of loans and advances from the Centre has declined from just over 57% of all State liabilities in 1991 to
6.6% in 2014. The fundamental reason for the decline in Central loans to States is the recommendation by
12th Finance commission in 2004-05 for the disintermediation of the Central Government from the raising
of public debt by State Governments. This recommendation led to the elimination of the loan portion of
Central plan transfers to States, as the Central Government since 2007-08 only makes grants to States
under plan transfers. The remainder of the plan funds must be raised by States themselves through, for
example, market borrowings. Hence, statement 3 is not correct.

15 www.visionias.in ©Vision IAS


Q 40.B
• Pair 1 is not correctly matched: Pochampally Ikat is a well-known form of saree made in Bhoodan
Pochampally and surrounding villages in Telangana State, India. these are popular for their traditional
geometric patterns with the Ikat style of dyeing. Pochampally Ikat saree is the first traditional Indian craft
to receive the status of Geographical Indication (GI) protection. Pochampally village has made it to
UNESCO tentative list of world heritage sites under the "iconic saree weaving clusters of India".
Pochampally Ikat is also known locally as Chitki, Pogudubandhu, and Buddabhashi in the region of
Telangana where it is produced, whereas in other parts of India, the handloom design is popularly known
as Pochampally. The fabric used is cotton, silk, and sico, which is actually a mix of silk and cotton. Ikat
represents a weaving form wherein the warp, weft and even both are tie-dyed before they are weaved to
create any designs on the finished fabric.
• Pair 2 is correctly matched: Thewa is a special art of jewelry making which involves fusing of
intricately worked-out sheet gold on molten glass. It evolved in Pratapgarh district, Rajasthan
India. Thewa is an art of fusing 23ct gold with multicoloured glass. It is a very detailed and intricate
process. A 23ct gold piece is first beaten into a very thin sheet. Intricate designs are inscribed onto these
gold sheets using very fine chisel. This gold sheet, called as “Thewa Ki Patti” is fixed to a lac-resin
compound spread on a board by slightly warming the lac and then pressing the gold sheet onto it. An open
work pattern is pierced thru these gold sheets placed on the lac-resin covered board by knocking off the
portions which ultimately creates the intricate design. The gold sheet is gently peeled off by heating it. It
has been accorded GI tag.
• Pair 3 is not correctly matched: Kani Shawl weaving originated in Kanihamal area of Kashmir
valley. Jammu & Kashmir's Kani shawls are made from wooden needles on a traditional handloom, it has
a very distinguishable, Mughal pattern that is woven into the fabric. Since Kani shawls are associated with
the best of handmade pashmina wool weaving, they have a high value in the market. They have been
accorded GI tag.

Q 41.A
• Exit poll is a post-voting poll, which is conducted just after a voter walks out after casting his or her vote.
Such polls aim at predicting the actual result on the basis of the information collected from voters. They
are conducted by a number of organizations. The basic step to predict exit polls is sampling.
• An opinion poll, sometimes simply referred to as a poll, is a kind of voter behavior survey conducted to
gauge the public opinion before voting takes place, while an exit poll happens right after voting.
• By a recent set of Guidelines issued, the Election Commission has stipulated that:
o the results of opinion polls can not be published between two days before the start of polling and after
the close of poll in any of the constituencies. Hence statement 2 is not correct.
o the results of exit polls can only be published or made otherwise known only after half an hour of the
end of polling hours on the last day of poll.
o EC exercises its power under section 126(1)(b) of Representation of Peoples Act, 1951, to ban the exit
polls. Hence statement 1 is correct.
o In November 2018, the Commission imposed a month-long ban on holding exit polls in the 5 poll-
bound states- Rajasthan, Madhya Pradesh, Chattisgarh, Mizoram and, Telangana.
o Reason for limited ban on exit polls - Many national and state parties have argued that these polls are
unscientific and biased. Political parties complained that these polls were sponsored by the opposition
parties and this may have a distorted effect on the mentality of the voters.

Q 42.A
• Accelerating Growth of New India’s Innovations or AGNIi is a national initiative under the guidance
and support of the Principal Scientific Adviser to the Government of India. It is being handled by
Invest India.
• AGNIi provides a platform for innovators to scale up their market-ready products by creating
pathways for licensing, technology transfer and market access. Further, AGNIi collaborates closely
with other stakeholders in the innovation ecosystem to support and augment their innovation and
technology commercialization initiatives. However, AGNIi is not a funding agency and does not
provide direct financial support to innovators. Hence option (a) is the correct answer.
• About Invest India: Invest India, set up in 2009, is a non-profit venture under the Department for
Promotion of Industry and Internal Trade, Ministry of Commerce and Industry, Government of India.
o It is the National Investment Promotion and Facilitation Agency of India and acts as the first point of
reference for investors in India.

16 www.visionias.in ©Vision IAS


o It focuses on sector-specific investor targeting and the development of new partnerships to enable
sustainable investments in India.
o In addition to a core team that focuses on sustainable investments, Invest India also partners with
substantial investment promotion agencies and multilateral organizations.
o Invest India also actively works with several Indian states to build capacity as well as bring in
global best practices in investment targeting, promotion and facilitation areas.

Q 43.B
• Supernova refers to both a type of star and a stellar explosion. A supernova is a star that suddenly
increases greatly in brightness because of a catastrophic explosion that ejects most of its mass.
• A supernova should not be confused with a quasar or quasars, which are believed to be the most
distant objects yet detected in the universe. Quasars give off enormous amounts of energy - they can be a
trillion times brighter than the Sun. Quasar is derived from a quasi-stellar radio source (star- like radio-
wave source), a term used by astronomers. In addition to radio waves and visible light, quasars also emit
ultraviolet rays, infrared waves, X-rays, and gamma-rays. Most quasars are larger than our solar system. A
quasar is approximately 1 kiloparsec in width.
• Quasars are believed to produce their energy from massive black holes in the center of the galaxies in
which the quasars are located. Because quasars are so bright, they drown out the light from all the other
stars in the same galaxy.
• Even if a quasar is very far away, astronomers can detect it if a galaxy closer to Earth acts as a lens and
makes the quasar look extra bright and big. The gravitational field of the closer galaxy warps space itself,
bending and amplifying the distant quasar’s light. This effect is called gravitational lensing. While
gravitational lensing is the phenomenon, A gravitational lens is a region of space containing a massive
object whose gravitational field distorts electromagnetic radiation passing through it in a similar way to a
lens, sometimes producing multiple images of a remote object. Hence X stands for gravitational lensing
and Y stands for quasars.
• Neutron Star: It is a celestial object of very small radius (typically 30 km) and very high density,
composed predominantly of closely packed neutrons. Neutron stars are thought to form by the
gravitational collapse of the remnant of a massive star after a supernova explosion, provided that the star
is insufficiently massive to produce a black hole.
• White Dwarf: It is a small very dense star that is typically the size of a planet. A white dwarf is formed
when a low-mass star (like our Sun) has exhausted all its central nuclear fuel and lost its outer layers as a
planetary nebula. Only the hot core of the star remains. This core becomes a very hot white dwarf, with a
temperature exceeding 100,000 Kelvin.
• Pulsar: It is a celestial object, thought to be a rapidly rotating (and highly magnetised) neutron star, that
emits regular pulses of radio waves and other electromagnetic radiation at rates of up to one thousand
pulses per second.

Q 44.C
• Statement 1 is correct: During the period of Vijayanagara, guilds played an important role in the
commercial life of the people and towns. The guilds were assemblies of merchants of the town. It was a
free and voluntary association of individuals. During the Vijayanagara period there were guilds of various
professions such as goldsmiths guild, weavers guild, cloth dyers guild, oil millers guild, potters guild,
cobblers guild, barbers guild and washermen guild etc. The craftsmen, as well as the mercantile groups in
medieval South India, were organized in corporations. Economically they were very sound. They used to
give loans on interest to their respective members. As the guilds had close contact with different people
they played an important role in the administration of the local area. They derived income from different
sources such as contributions from the members, profit from the banking operations, income from the
corporate property etc. These different guilds were representative in character and safeguarded the
interests of the local people. The officials of the state never interfered with the rights of these guild
assemblies. The prosperity of the guilds encouraged them to make liberal grants to temples and they also
constructed tanks.The Pattanasvami was the head of the mercantile community of a town. He levied
and collected the revenues at the time of fairs. And during the market days, he collected taxes from
different shops and from merchants. The guilds enjoyed certain powers to levy local taxes and they
spent the collected taxes for some good purpose.
• The Vijayanagara empire for the first time in medieval times developed good contacts with foreign
merchants like Portuguese and Arab. Barbosa in his account has made a good narration of the foreign
trade how it passed from the hands of Muslims to the Portuguese. With the coming of Portuguese towards

17 www.visionias.in ©Vision IAS


the close of the 15th century, this monopoly of the Muslim traders was broken. Even during the time of
Praudhadevaraya till about the commencement of the sixteenth century, the horse trade was largely in the
hands of Muslim traders. Bhatkal, Honnavara, Mangalore and Goa were the important harbors or ports at
that time. The Ormuz ships were coming to Bhatkal with horses and pearls. Other imports were elephants,
copper, coral, mercury, vermilion, china silk, and velvet. Elephants were imported from Ceylon and silk
from China.
• Statement 2 is not correct: Taxes were collected from foreign merchants, which was also a source of
income to the state. Articles like cloth, rice, iron, saltpeter, sugar, and spices were exported to Portugal.
Some of the important port towns were Ankola, Calicut, Basrur, Barakur and Mangalore. Foodgrains were
exported to Maldives Islands, Ormuz and Aden. Iron ore was purchased by the Portuguese. Sugar was
exported to Ormuz. Ships came from Mecca to Bhatkal for spices. Foreign trade was also there with
Malacca, Pegu, and Sumatra. Thus, foreign trade attracted a large number of merchants, both Indian
and foreign merchants to purchase their required materials from the Vijayanagara market. Foreign
trade greatly enriched the Vijayanagara Empire.
• Statement 3 is not correct: During the Vijayanagara period, gold and copper coins seem to be more in
circulation than silver coins. The silver coins did not come into popular use. The reason was that in
the empire there were several gold and copper mines, but silver was not available and had to be purchased
for the issue of the coin. A separate department of mints was established, which exercised control over
minting operations. Officers were appointed to supervise the royal mints. An important aspect of the
coins of Vijayanagara is the variety of coins of reverse and obverse. They had legends on the reverse
and the obverse was reserved for gods and goddesses. Coins shed much light on socio-political and
economic history as well playing a subsidiary role in understanding how art was used. The issue of
currency of only some particular denominations was monopolized by the state.
• Additional Information:
o Salt manufacturing industry was one of the most flourished industries at that time. Places like
Honnavara, Bhatkal, Mangalore, Barakur, Ankola were not only important port centers but also
leading salt producing places in the Vijayanagara empire, through which foreign trade was carried out.
Salt manufactured in the coastal region was supplied to the interior towns by local merchants. Salt
manufacture appears to have been a state monopoly. The right to manufacture salt at different places
seems to have been leased by the state to the highest bidders.
o The taxation system was quite progressive. On the goods which were not luxury and equally used
by all communities the tax levied was same, while for luxury goods, the taxes were greater for rich
classes but lower for poor classes.
o There was a defined system of taxation on industries in the Vijayanagara Kingdom. Several
industries were monopolized, while several others were left open for all. The industries were levied
taxes according to the production and demand for the goods. One of the best available examples is the
tax on oil pressing industries. The oil crushing industry was subjected to a number of taxes levied by
the state. Epigraphs of the period refer to the taxes levied on the oil-mill known as Ganasidhaya. It
was also a practice that each oil mill was required to bear an official stamp and for offering this stamp
the government collected a tax.
Q 45.B
• The provisions of Part IX of the constitution relating to the Panchayats are not applicable to the Fifth
Schedule areas. However, the Parliament may extend these provisions to such areas, subject to such
exceptions and modifications as it may specify. Under this provision, the Parliament has enacted the
“Provisions of the Panchayats (Extension to the Scheduled Areas) Act”, 1996, popularly known as the
PESA Act or the Extension Act.
• The objectives of the PESA Act are as follows:
o To extend the provisions of Part IX of the Constitution relating to the panchayats to the scheduled
areas with certain modifications
o To provide self-rule for the bulk of the tribal population
o To have village governance with participatory democracy and to make the gram sabha a nucleus of all
activities
o To evolve a suitable administrative framework consistent with traditional practices
o To safeguard and to preserve the traditions and customs of tribal communities
o To empower panchayats at the appropriate levels with specific powers conducive to tribal
requirements
o To prevent panchayats at the higher level from assuming the powers and authority of panchayats at
the lower level of the Gram Sabha. Hence option (d) is correct.

18 www.visionias.in ©Vision IAS


• The Act says that the State Legislature shall endeavour to follow the pattern of the Sixth Schedule to the
Constitution while designing the administrative arrangements in the Panchayats at district levels in the
Scheduled Areas. Hence option (c) is correct.
• As per the act, every village shall have a Gram Sabha consisting of persons whose names are included in
the electoral rolls for the Panchayat at the village level. Hence option (a) is correct.
• The reservation of seats in the Scheduled Areas at every Panchayat shall be in proportion to the
population of the communities in that Panchayat for whom the reservation is sought to be given under Part
IX of the Constitution; Provided that the reservation for the Scheduled Tribes shall not be less than one-
half of the total number of seats; Provided further that all seats of Chairpersons of Panchayats at all levels
shall be reserved for the Scheduled Tribes; Hence option (b) is not correct.

Q 46.C
• The initial Turkish conquests in India in the early 13th century displaced many local chiefs. In order to
consolidate, the Turkish rulers made revenue assignments (iqta), in lieu of cash, to their nobles. So,
Iqtas were provinces or spheres of influence that were put under the charge of officers called
'Iqtadars' (governors), also known as Muqtis or Walis. Hence statement 1 is not correct.
o lqta is an Arabic word and the institution had been in force in the early lslamic world as a form of
reward for services to the state. It was used in the Caliphate administration as a way of financing
operations and paying civil and military officers. It was not an ancient indigenous institution.
o The grant of iqta did not imply a right to the land nor was it hereditary. Though the holders of
iqta tended to acquire hereditary rights in Feroz Tughluq's reign. These revenue assignments were
transferable, the iqta-holder being transferred from one region to another every three or four years.
• Iqtas later became provinces or Subas. Initially, Muqtis were almost independent and were expected to
maintain law and order and collect the land revenue in their tracts. However, as Central Government
became stronger it began to control the Muqtis more closely and the salaries of Muqtis were fixed in cash.
The Muqtis were required to remit to the center the balance of the income after meeting the expenditure.
Their accounts were audited with harshness.
• Below the provinces, there were 'Shiqs' and below them, the 'Paragnas' and below them were the villages.
• Shiq was like the districts in the current era.
• The villages were grouped into units of 100 or 84 which formed the Parganas. The important officials of
a pargana were the amil, the mushrif also known as amin or munsif the treasurer, the qanungo and
two karkuns (clerks). The pargana was an important administrative unit because it was there that
the government came into direct contact with the peasants. Hence statement 2 is not correct.
• The smallest unit of administration was the village which was administered by local hereditary officers
and the panchayat of the village. The Chaudhri, the Patwari, the Khut, the Muqaddam and the
Chaukidar were the hereditary officers of the village who helped the government in collection of the
revenue and enjoyed certain privileges except during the reign of Ala-ud-din Khalji. The Panchayat
of the village looked after education, sanitation, etc. and acted as a judicial body as well concerning
disputes in the village. Khuts were small landowners in villages. Hence statement 3 is correct.
Q 47.D
• Copyright is a right given by the law to creators of literary, dramatic, musical and artistic works and
producers of cinematograph films and sound recordings. In fact, it is a bundle of rights including, inter
alia, rights of reproduction, communication to the public, adaptation, and translation of the work. There
could be slight variations in the composition of the rights depending on the work. Copyright ensures
certain minimum safeguards of the rights of authors over their creations, thereby protecting and rewarding
creativity.
• Copyright refers to a bundle of exclusive rights vested in the owner of copyright by virtue of Section 14 of
the Copyright Act, 1957. These rights can be exercised only by the owner of the copyright or by any other
person who is duly licensed in this regard by the owner of copyright. These rights include the right of
adaptation, right of reproduction, right of publication, right to make translations, communication to public
etc.
• The Copyright Act, 1957 protects original literary, dramatic, musical and artistic works and
cinematograph films and sound recordings from unauthorized uses. Unlike the case with patents,
copyright protects the expression and not the ideas. There is no copyright in an idea. Hence
statement 2 is correct.
• Copyright does not ordinarily protect titles by themselves or names, short word combinations, slogans,
short phrases, methods, plots or factual information. Copyright does not protect ideas or concepts. To get
the protection of copyright a work must be original.

19 www.visionias.in ©Vision IAS


• Acquisition of copyright is automatic and it does not require any formality. Copyright comes into
existence as soon as a work is created and no formality is required to be completed for acquiring
copyright. However, certificate of registration of copyright and the entries made therein serve as prima
facie evidence in a court of law with reference to dispute relating to ownership of copyright. Any
individual who is an author or rights owner or assignee or legal heir can file application for copyright of a
work either at the copyright office or by post or by e-filing facility from the copyright Office web-site
"www.copyright.gov.in". Hence statement 1 is correct.
• Copyright protection earlier came under the purview of Ministry of Human Resource Development, but
was later shifted under Department For Promotion of Industry and Internal Trade, Ministry of Commerce
and Industry. Hence, statement 3 is correct.

Q 48.D
• Coevolution, the process of reciprocal evolutionary change that occurs between pairs of species or among
groups of species as they interact with one another. The activity of each species that participates in the
interaction applies selection pressure on the others.
• These ecological relationships include:
o Predator/prey and parasite/host
o Competitive species
o Mutualistic species
• Predator/prey and parasite/host: In a predator-prey interaction, for example, the emergence of faster
prey may select against individuals in the predatory species who are unable to keep pace. Thus, only fast
individuals or those with adaptations allowing them to capture prey using other means will pass their
genes to the next generation. Both intraspecific competition (whereby members of the same species
compete for limited resources), with features such as sexual conflict and sexual selection, and interspecific
competition (in which individuals of different species compete for the same resources), such as between
predators, may be able to drive coevolution.
• Mutualistic species: Plants and insects represent a classic case of mutualistic coevolution. Many plants
and their pollinators are so reliant on one another and their relationships are so exclusive that biologists
have good reason to think that the "match" between the two is the result of a coevolutionary process.
• Commensalism: An association between two organisms in which one benefits and the other derives
neither benefit nor harm. It does not involve much physiologic interaction or dependency between the two
partners, the host and the commensal. The two partners can survive independently. It does not promote
coevolution. Hence option (d) is correct.

Q 49.B
• National Company Law Appellate Tribunal (NCLAT) was constituted under Section 410 of the
Companies Act, 2013 for hearing appeals against the orders of National Company Law Tribunal
(NCLT). Hence, statement 1 is not correct.
• NCLAT consisting of a chairperson and such number of Judicial and Technical Members, not exceeding
eleven, as the Central Government may deem fit. The chairperson shall be a person who is or has been a
Judge of the Supreme Court or the Chief Justice of a High Court. The chairperson and Judicial Members
of the Appellate Tribunal shall be appointed after consultation with the Chief Justice of India.
• NCLAT is also the Appellate Tribunal for hearing appeals against the orders passed by NCLT under
Section 61 of the Insolvency and Bankruptcy Code, 2016 (IBC).
• NCLAT is also the Appellate Tribunal for hearing appeals against the orders passed by Insolvency and
Bankruptcy Board of India under Section 202 and Section 211 of IBC.
• NCLAT is also the Appellate Tribunal to hear and dispose of appeals against any direction issued or
decision made or order passed by the Competition Commission of India (CCI). Hence, statement 2
is correct.
• Any person aggrieved by any order of the Appellate Tribunal may file an appeal to the Supreme
Court within 45 days from the date of receipt of the order of the Appellate Tribunal to him on any
question of law arising out of such order. Hence, statement 3 is correct.

Q 50.B
• Standing bodies of fresh water are often divided into categories that reflect levels of biological
production.
• An oligotrophic lake or water body is one which has relatively low productivity due to the low nutrient
content in the lake. These lakes usually have quite clear water due to the limited growth of algae in the

20 www.visionias.in ©Vision IAS


lake. The waters of such lakes are of high-drinking quality. Such lakes support aquatic species who
require well-oxygenated, cold waters such as lake trout fish. Oligotrophic lakes are usually found in the
cold regions of the world where mixing of nutrients is rare and slow due to the low temperatures of the
lake waters.
• Oligotrophic lakes have the following characteristics:
o Net primary production is only between 50 and 100 milligrams of carbon per square metre per
day (very low),
o nutrients are in poor supply, and
o secondary production is depressed.
• On the other hand, Eutrophic lakes are productive, net primary production is between 600 and 8,000
milligrams of carbon per square metre per day, nutrients are in good supply, and secondary production is
high.
• Mesotrophic lakes are lakes of intermediate productivity, net primary production is between 250 and
1,000 milligrams of carbon per square metre per day.

Q 51.C
• In March 1942, a mission headed by Stafford Cripps was sent to India with constitutional proposals to
seek Indian support for the war. Stafford Cripps was a left-wing Labourite, the leader of the House of
Commons and a member of the British War Cabinet who had actively supported the Indian national
movement. The mission was sent to India during the tenure of Viceroy Linlithgow. Hence, option (b)
is correct.
• The main proposals of the mission were as follows.
o An Indian Union with a dominion status would be set up; it would be free to decide its relations with
the Commonwealth and free to participate in the United Nations and other international bodies.
o After the end of the war, a constituent assembly would be convened to frame a new constitution.
Members of this assembly would be partly elected by the provincial assemblies through proportional
representation and partly nominated by the princes. However, the idea of Constituent Assembly was
mooted by the August Offer of 1940 for the first time. Hence, option (c) is not correct.
o The British government would accept the new constitution subject to two conditions:
✓ any province not willing to join the Union could have a separate constitution and form a separate
Union.
✓ the new constitution-making body and the British government would negotiate a treaty to effect
the transfer of power and to safeguard racial and religious minorities.
o In the meantime, defence of India would remain in British hands and the governor-general’s powers
would remain intact.
• The Mission led by Sir Stafford Cripps in the early 1942 had offered what Gandhi quipped as a ‘post
dated cheque on a bank largely failing’. The sole purpose of Cripps Mission was to convince India to
support the British war by supplying the wherewithal to their army, soldiers in particular. The Dominion
status it promised India in return after the war was quite vague and without a definite time frame. Indian
Leaders felt deceived when the Mission announced that the British would hold complete control over
Indian affairs in the mean time. Hence, option (a) is correct.

Q 52.C
• Article 361B: A member of a House belonging to any political party who is disqualified for being a
member of the House under paragraph 2 of the Tenth Schedule shall also be disqualified to hold any
remunerative political post for the duration of the period commencing from the date of his disqualification
till the date on which the term of his office as such member would expire or till the date on which he
contests an election to a House and is declared elected, whichever is earlier. Hence, statement 1 is
correct.
• Explanation: For the purposes of this article-
o the expression “House” has the meaning assigned to it in clause (a) of paragraph 1 of the Tenth
Schedule;
o the expression “remunerative political post” means any office:
✓ under the Government of India or the Government of a State where the salary or remuneration for
such office is paid out of the public revenue of the Government of India or the Government of the
State, as the case may be; or

21 www.visionias.in ©Vision IAS


✓ under a body, whether incorporated or not, which is wholly or partially owned by the Government
of India or the Government of State, and the salary or remuneration for such office is paid by such
body;
o except where such salary or remuneration paid is compensatory in nature. Hence, Statement 2 is
correct.
• Office of Profit: It is a position in the government that cannot be held by an MLA or an MP. The post can
yield salaries, perquisites and other benefits. The origin of this term can be found in the English Act of
Settlement, 1701. Under this law, "no person who has an office or place of profit under the King, or
receives a pension from the Crown, shall be capable of serving as a member of the House of Commons."
This was instituted so that there wouldn't be any undue influence from the royal household in
administrative affairs.
o According to Articles 102(1)(a) and 191(1)(a) of the Constitution, an MP or MLA is barred from
holding an office of profit as it can put them in a position to gain a financial benefit. "A person shall
be disqualified for being chosen as, and for being, a member of either House of Parliament, (a) if he
holds any office of profit under the Government of India or the Government of any State, other than
an office declared by Parliament by law not to disqualify its holder," says the law.
o Under the Representation of People Act too, holding an office of profit is grounds for disqualification.
o The law does not clearly define what constitutes an office of profit but the definition has evolved over
the years with interpretations made in various court judgments. An office of profit has been
interpreted to be a position that brings to the office-holder some financial gain, or advantage, or
benefit. The amount of such profit is immaterial.
o In 1964, the Supreme Court ruled that the test for determining whether a person holds an office of
profit is the test of appointment. Several factors are considered in this determination including factors
such as:
✓ whether the government is the appointing authority,
✓ whether the government has the power to terminate the appointment,
✓ whether the government determines the remuneration,
✓ what is the source of remuneration, and
✓ the power that comes with the position.
o Provisions of Articles 102 and 191 also protect a legislator occupying a government position if the
office in question has been made immune to disqualification by law. In the recent past, several state
legislatures have enacted laws exempting certain offices from the purview of office of profit.
Parliament has also enacted the Parliament (Prevention of Disqualification) Act, 1959, which has been
amended several times to expand the exempted list.

Q 53.C
• The Common Risk Mitigation Mechanism under the International Solar Alliance seeks to de-risk
and reduce the financial cost of solar projects in ISA-member countries.
• CRMM will act as a pooled insurance with limited liability. Banks and multi-lateral institutions can
contribute to the fund for a marginal premium. This will lower the cost of capital for developing
renewable energy projects.
• The CRMM will offer a simple and affordable tool that will create a secure environment for private
institutional investment in solar assets. The instrument will help diversify and pool risks on mutualized
public resources and unlock significant investments.
• Indian corporates have already given $7 million to the CRMM. Coal India, Power Finance Corporation
and ITPO (India Trade Promotion Organisation) will contribute another $1 million each to the CRMM.

Q 54.A
• Statement 1 is correct: As the name implies, end-to-end encryption protects data such that it can
only be read on the two ends, by the sender, and by the recipient. No one else can read the
encrypted data, including hackers, governments, and even the server through which the data is
passing. End-to-end encryption is based on asymmetric encryption. Under Asymmetric encryption, two
types of keys are used for each party, one public key and one private key. The public keys are available to
both parties, and to anyone else, as the two parties mutually share their public keys prior to
communication. This private key is only available to the receiver and to no one else, not even to the
sender. This key is the one element that makes it impossible for any other party to decrypt the message
because there is no need to send the private key over.

22 www.visionias.in ©Vision IAS


• Statement 2 is not correct: For many other services that offer encryption, the data is encrypted during
transfer but is protected only from outside intruders like hackers. The service can intercept the data at their
servers and use them. They can potentially hand the data to third parties or to law enforcement
authorities. Thus, even if they want to, the service cannot intercept and do anything with the data.
Law enforcement authorities and governments are also among those who cannot access the data,
even with authorization. Theoretically, no one can, except the parties at the two ends.

Q 55.A
• The Barabar Hill Caves are the oldest surviving rock-cut caves in India, dating from the Maurya Empire
(322–185 BCE), some with Ashokan inscriptions, located in the Makhdumpur region of Jehanabad
district, Bihar, India, 24 km (15 mi) north of Gaya.
• These caves are situated in the twin hills of Barabar (four caves) and Nagarjuni (three caves); caves of the
1.6 km (0.99 mi)-distant Nagarjuni Hill are sometimes singled out as the Nagarjuni Caves. These rock-
cut chambers bear dedicatory inscriptions in the name of "King Piyadasi" for the Barabar group,
and "Devanampiya Dasaratha" for the Nagarjuni group, thought to date back to the 3rd century
BCE during the Maurya period, and to correspond respectively to Ashoka (reigned 273–232 BCE)
and his grandson, Dasharatha Maurya.
• The caves were used by ascetics from the Ajivika sect, founded by Makkhali Gosala. The Ajivikas
had many similarities with Buddhism as well as Jainism.
• Barabar Hill contains four caves: Karan Chaupar, Lomas Rishi, Sudama and Viswakarma.
• The nearby caves of Nagarjuni hill were built few decades later than the Barabar caves, and consecrated
by Dasaratha Maurya, Ashoka's grandson and successor, each for the Ajivikas sect. They are 1.6
kilometers east of the Barabar Caves. The three caves are: Gopika, Vadithi-ka-Kubha, Vapiya-ka-
Kubha.

Q 56.D
• Currency depreciation is a fall in the value of a currency in a floating exchange rate system. Currency
depreciation can occur due to factors such as economic fundamentals, interest rate differentials, political
instability or risk aversion among investors.
• The rupee depreciation may feed into inflation by affecting the price of imported goods, especially
oil. Depreciation currently brings many costs in lieu of a few benefits.
• Causes of Rupee depreciation:
o Uncertainty with regard to global output and trade growth was the basic factor responsible for the
deceleration and reversal in the volume of net portfolio investment inflows. Thus weakening rupee.
o Evidence that the Indian economy has turned sluggish has not helped an already weakened investor
sentiment.
o The increase in External Commercial Borrowings (ECBs) that resulted from the policy changes has
also increased the exposure of Indian business to foreign currency risk.
• Following steps taken can be taken by the RBI and the government of India to stabilize the currency
markets:
o Allowing higher limits of outflows creates more demand for foreign currency and thus depreciating
rupee much further. RBI can reduce the limit under the Liberalised Remittance Scheme (LRS). These
strong steps can safeguard the currency from depreciating much further. Hence, statement 1 is not
correct.
o Increasing customs duty would increase prices of gold in the domestic market and thus lowers the
demand for gold. Lower demand for gold would lower the gold imports. Thus less foreign currency is
required for trade and currency stabilizes. The Finance Ministry (Government) can increase the
customs duty on importing precious metals including gold and platinum. Hence, statement 2 is not
correct.
o Japan and India entered into a $75-billion currency swap arrangement that will bolster the country’s
firepower as it battles a steep drop in the rupee’s value. This New Swap Agreement should aid in
bringing greater stability to foreign exchange & capital markets in India.
o Removed administrative restrictions on investment schemes offered by banks to non-resident Indians
would increase foreign investment inflows. RBI has removed administrative restrictions and the
ceiling on interest rates on deposit accounts held by NRIs. The government liberalized the FDI limits
for 12 sectors, including oil and gas. This would increase foreign investment inflows and thus
stabilize Indian rupee. Hence, statement 3 is not correct.

23 www.visionias.in ©Vision IAS


Q 57.A
• Plasma technology is based on a simple physical principle. Matter changes its state when energy is
supplied to it: solids becomes liquid, and liquids becomes gaseous. If even more energy is supplied to a
gas, it is ionized and goes into the energy-rich plasma state, the fourth state of matter.
• When plasma comes into contact with solid materials like plastics and metals, its energy acts on the
surfaces and changes important properties, such as the surface energy.
• Applications:
o Plasma treatment is most often used to clean, enhance the adhesion qualities of surfaces and produce
thin coatings.
o Plasma technology is a dry, cleanroom-compatible, environmentally friendly alternative to
various conventional solvent-based chemical surface treatment methods. In addition, this technology
uses no water and fewer chemicals.
o Plasma coatings are particularly appropriate for textile-made biomaterials to improve their
biocompatibility or improve (or reduce) biological activity on the surface. Among different medical
textile applications, surface activation confers hydrophilic properties on wovens and nonwovens used
in blood filtration devices or in body fluid filtration membranes. Other applications include plasma
grafting for growth test fabrics, fermentation membranes, implants, catheter fabrics, enzyme-
immobilization substrates and textile grafts.
o Plasma gasification is an emerging technology which can process landfill waste to extract
commodity recyclables and convert carbon-based materials into fuels. It can form an integral
component in a system to achieve zero-waste and produce renewable fuels, whilst caring for the
environment.
o Oils spills can be cleaned through plasma polymeriation which is surface modification using
plasma. Recently a team of researchers from Argentine developed a highly hydrophobic with high oil
adsorption capacities plasma‐polymerized carbonaceous nanosponge (CN). It consists of a
powder‐like agglomeration of nanoparticles conforming a porous micrometric structure. Selective
sorption of the CN makes them suitable in various applications, including the areas of water filtration,
liquid separation and hydrocarbons spill cleanup.
o Plasma Medicine is a growing field that is having an impact in several important areas in therapeutic
patient care, combining plasma physics, biology, and clinical medicine. The most active areas of
plasma technology applications are in wound treatment; tissue regeneration; inactivation of
pathogens, including biofilms; treating skin diseases; and sterilization.

Q 58.A
• The Intergovernmental Science-Policy Platform on Biodiversity and Ecosystem Services (IPBES) is
an independent intergovernmental body established by States to strengthen the science-policy interface
for biodiversity and ecosystem services for the conservation and sustainable use of biodiversity,
long-term human well-being and sustainable development.
• It was established in Panama City, on 21 April 2012 by 94 Governments. It is not a United Nations
body. However, at the request of the IPBES Plenary and with the authorization of the UNEP Governing
Council in 2013, the United Nations Environment Programme (UNEP) provides secretariat services
to IPBES.
• IPBES currently has over 134 member States. All States Members of the United Nations are eligible for
IPBES membership but not all UN states are its members. A large number of NGOs, organizations,
conventions and civil society groupings also participate in the formal IPBES process as observers. Hence
statement 2 is not correct.
• The work of IPBES can be broadly grouped into four complementary areas:
o Assessments: On specific themes (e.g. “Pollinators, Pollination and Food Production”);
methodological issues (e.g. “Scenarios and Modelling); and at both the regional and global levels (e.g.
“Global Assessment of Biodiversity and Ecosystem Services”).
o Policy Support: Identifying policy-relevant tools and methodologies, facilitating their use, and
catalyzing their further development.
o Building Capacity & Knowledge: Identifying and meeting the priority capacity, knowledge and
data needs of our member states, experts and stakeholders for the conservation and sustainable
use of biodiversity. Hence statement 1 is correct.
o Communications & Outreach: Ensuring the widest reach and impact of our work.
• IPBES currently has over 134 member States. A large number of NGOs, organizations, conventions and
civil society groupings also participate in the formal IPBES process as observers, with several thousand

24 www.visionias.in ©Vision IAS


individual stakeholders, ranging from scientific experts to representatives of academic and research
institutions, local communities and the private sector, contributing to and benefiting from our work.
• India is its member since its inception in 2012.

Q 59.C
• A person to be eligible for election as President should fulfill the following qualifications:
o He should be a citizen of India.
o He should have completed 35 years of age.
o He should be qualified for election as a member of the Lok Sabha.
o He should not hold any office of profit under the Union government or any state government or any
local authority or any other public authority.
o A sitting President or Vice-President of the Union, the Governor of any state and a minister of the
Union or any state is not deemed to hold any office of profit and hence qualified as a presidential
candidate.
• To be eligible for election as Vice-President (or the Chairman of the Rajya Sabha), a person should
fulfill the following qualifications:
o He should be a citizen of India.
o He should have completed 35 years of age.
o He should be qualified for election as a member of the Rajya Sabha.
o He should not hold any office of profit under the Union government or any state government or any
local authority or any other public authority.
• The Speaker is elected by the Lok Sabha from amongst its members (as soon as may be, after its first
sitting). The Constitution lays down the following qualifications for a person to be chosen as a member of
Parliament (Both Lok Sabha and Rajya Sabha):
o He must be a citizen of India.
o He must make and subscribe to an oath or affirmation before the person authorized by the election
commission for this purpose. In his oath or affirmation, he swears
✓ To bear true faith and allegiance to the Constitution of India
✓ To uphold the sovereignty and integrity of India
o He must be not less than 30 years of age in the case of the Rajya Sabha and not less than 25 years of
age in the case of the Lok Sabha.
o He must possess other qualifications prescribed by Parliament.
• The Prime Minister of India if not a member of either of the two houses of the Parliament has to become a
member within six months of joining the office. There is no separate age criterion for the office of Prime
Minister.
• Thus for the offices of the President and the Vice-President, a Member of the Parliament (MP) needs to
meet the age criteria of 35 years. Hence only options 1 and 3 are correct.

Q 60.B
• The Kishangarh province in Rajasthan is known for its Bani Thani paintings. It is a totally different
style with highly exaggerated features like long necks, large, almond shaped eyes, and long fingers. This
style of painting essentially depicts Radha and Krishna as divine lovers, and beautifully portrays their
mystical love. Kishangarh miniature painting reached a peak in the eighteenth century, during the
rule of Raja Sawant Singh, who fell in love with a slave girl, Bani Thani and commanded his artists
to portray himself and her as Krishna and Radha. Other themes of Bani Thani paintings include
portraits, court scenes, dancing, hunting, music parties, nauka vihar (lovers travelling in a boat), Krishna
Lila, Bhagavata Purana and various other festivals like Holi, Diwali, Durga puja, and Dussehra. Hence
pair 1 is correctly matched.
• Thangka is a typical painting of Sikkim. Thangka created on cotton canvas and often framed with silk.
They depict images of different deities and philosophies related to Buddhism. Traditionally made by the
budhist monks and priests and specific ethnic groups. Practitioners use thangkas to develop a clear
visualization of a particular deity, strengthening their concentration, and forging a link between
themselves and the deity. Hence, pair 2 is correctly matched. There are 3 types of Thangka:
o One depicts life of Budhha, his birth, his search for enlightenment and understanding of life.
o The second is Buddhist belief of life and death.
o Third type, for their use as aids in meditational practices.
• Thanjavur or Tanjore has a unique place in the history of Indian painting, in that it houses the 11th
century Chola wall paintings in the Brihadeeswarar temple (Periya koyil or Pervudaiyar koyil in Tamil) as

25 www.visionias.in ©Vision IAS


also paintings from the Nayak period (many times superimposed on the earlier Chola paintings) dating to
the 16th/17th century.
o Tanjore painting ranks among the greatest traditional art forms for which India is noted
worldwide. Their themes are fundamentally mythological. Originating in Tanjavur about 300 kms
from Chennai, this form of art developed at the height of cultural evolution achieved during the rule
of mighty Chola empire. The art form evolved and flourished under the patronage of successive
rulers.
o Chola murals were created by applying lime plaster first and then drawing over the plaster. But in
Tanjore painting of Nayak period, chalk powder or limestone powder was applied and then drawing
was made. Natural colors from plants and fruits were used in Chola and Nayak periods. So, the only
difference between Chola murals and Tanjore paintings of Nayak periods is Tanjore painting was
decorated with stones and gold foils.
o The characteristics of the Tanjore paintings are their brilliant colour schemes, decorative
jewellery with stones and cut glasses and remarkable gold leaf work. The liberal use of gold leaf
and precious and semi-precious stones presents a splendid visual treat. These give life to the
pictures such that the pictures come alive in a unique way. Adorned with rubies, diamonds and other
precious gemstones, and trimmed with gold foil, Tanjore paintings were true treasures.
o The subjects of most of these paintings are Hindu gods, goddesses, and saints, including
important episodes from the Hindu Puranas, and other religious texts. It mainly consists of
themes on Hindu gods and goddesses, with figures of Lord Krishna in various poses and depicting
various stages of his life being the favourite. Hence pair 3 is correctly matched.
o The Thanjavur paintings are actually panel paintings which are done on wooden planks, and
hence these are also referred to as 'palagai padam'. Thanjavur paintings have been recognized
with Geographical Indication (GI) rights by the Government of India in the year 2007.

Q 61.D
• The presiding officer in Rajyasabha (i.e the Vice-President) cannot preside over a sitting of the Rajya
Sabha as its Chairman when a resolution for his removal is under consideration. However, he can be
present and speak in the House and can take part in its proceedings, without voting, even at such a
time. He can also be removed from the office before the completion of his term. A formal impeachment is
not required for his removal. He can be removed by a resolution of the Rajya Sabha passed by a majority
of all the then members of the House (which is called the effective majority) and agreed to by the
Lok Sabha. But, no such resolution can be moved unless at least 14 days’ advance notice has been given.
Notably, no ground has been mentioned in the Constitution for his removal.
• The Presiding officer in Loksabha (i.e the Speaker) can vote in the first instance when a resolution
for his removal is under consideration of the Lok Sabha. The Speaker can be removed from office only
on a resolution of the House passed by a majority of all the then members of the House, i.e an
effective majority. Such a resolution has to satisfy some conditions like: it should be specific with
respect to the charges and it should not contain arguments, inferences, ironical expressions, imputations or
defamatory statements, etc. Not only these, but discussions should also be confined to charges referred to
in the resolution. It is also mandatory to give a minimum of 14 days' notice of the intention to move the
resolution.

Q 62.C
• The Pachmarhi Biosphere Reserve is located in the biogeographical region of the Deccan Peninsula and
the Biotic Province of Central India. The Satpura mountain ranges cross India from west to east and
Pachmarhi lies directly in its center.
• The eastern boundary of the biosphere reserve lies along a road with cultivation farms, close to the Dudhi
River, while the southern boundary borders the Tawa plateau.
• Pachmarhi comprises three protection sites:
o the Bori Sanctuary,
o Satpura National Park and
o Pachmarhi Sanctuary – otherwise known as the Satpura Tiger Reserve. Hence option (c) is correct.
• The Pachmarhi Plateau is also known as the ‘Queen of Satpura’ because it contains valleys, marshes,
streams, and waterfalls, all of which have led to the development of unique and varied biodiversity.
• Gangau wildlife sanctuary is part of Panna Tiger Reserve which also includes Panna National Park
and Ken Gharial Sanctuary.

26 www.visionias.in ©Vision IAS


Q 63.C
• Dumping is said to occur when the goods are exported by a country to another country at a price lower
than its normal value. This is an unfair trade practice that can have a distortive effect on international
trade. Anti-dumping is a measure to rectify the situation arising out of the dumping of goods and
its trade distortive effect. Thus, the purpose of anti-dumping duty is to rectify the trade distortive
effect of dumping and re-establish fair trade. The use of anti-dumping measures as an instrument of
fair competition is permitted by the WTO. Duty X is an anti-dumping duty as it imposed to counter
trade distortive effect caused by the dumping of goods below the normal price.
• Apart from dumping, some of the countries also resort to the subsidization of their exports to other
countries. Export subsidies, under the WTO agreement, are treated as unfair trade practice and such
subsidies are actionable by way of levy of anti-subsidy countervailing duty. Duty Y is a countervailing
duty as it imposed to counter the trade distortion caused by the subsidization of exports.
• There is one more trade remedial measure called "safeguards" which are applied as an emergency
measure in response to the surge in imports of a particular item. Safeguard duty is applied when:
o there is a surge in imports of a particular product irrespective of a particular country/ies and,
o it causes serious injury to the domestic industry.
• Duty Z is safeguard duty as it is imposed to curb the effects of a sudden increase in imports.
• Hence option (c)is the correct answer.

Q 64.B
• Side pocketing rule: Side pocketing is a framework that allows mutual funds to segregate the bad assets
in a separate portfolio within their debt schemes. Hence, option (b) is correct.
• It was introduced by the Securities and Exchange Board of India (SEBI) after the IL&FS fiasco — after it
emerged that many fund houses have huge exposure to the beleaguered entity and could potentially take a
huge hit on their net asset value thereby affecting investor returns.
• How does it work?
o The capital markets regulator has laid down that such a portfolio can be created only if there is a
credit event at the issuer level in the form of a downgrade of a debt or money market instrument to
'below investment grade' or subsequent downgrades from such levels.
o In other words, if a debt instrument is downgraded to default rating by credit rating agencies, then the
fund house has the option to create a side pocket so that good asset can be ring-fenced.
• Provisioning is a part of the RBI's prudential regulation norm. Under provisioning, banks have to set
aside or provide funds to a prescribed percentage of their bad assets. Hence, option (d) is not correct.

Q 65.D
• Statement 1 is correct: The First World War proved a boon for Indian Industrialists and gave Indian
industries some sort of protection against the foreign competition. Foreign imports fell to a very low
while there was an increase in government purchases for war purposes. Exports of raw material
became less which kept down their prices to the advantage of Indian industrialists. But wages did not rise.
• Statement 2 is not correct: In 1914 the First World War broke out. Steel produced in Britain now had
to meet the demands of the war in Europe. So imports of British steel into India declined
dramatically and the Indian Railways turned to TISCO for the supply of rails. As the war dragged on
for several years, TISCO had to produce shells and carriage wheels for the war. By 1919 the colonial
government was buying 90 percent of the steel manufactured by TISCO in India. Over time TISCO
became the biggest steel industry within the British empire.
• In the case of iron and steel, as in the case of cotton textiles, industrial expansion occurred only when
British imports into India declined and the market for Indian industrial goods increased. This happened
during the First World War and after.
• Statement 3 is correct: The first cotton mill in India was set up as a spinning mill in Bombay in 1854.
From the early nineteenth century, Bombay had grown as an important port for the export of raw cotton
from India to England and China. The growth of cotton mills led to a demand for labor. Thousands of
poor peasants, artisans and agricultural laborers moved to the cities to work in the mills.
• In the first few decades of its existence, the textile factory industry in India faced many problems. It found
it difficult to compete with the cheap textiles imported from Britain. In most countries, governments
supported industrialization by imposing heavy duties on imports. The colonial government in India
usually refused such protection to local industries. During the First World War, there was a first
major spurt in the development of cotton factory production in India, as textile imports from
Britain declined and Indian factories were called upon to produce cloth for military supplies.

27 www.visionias.in ©Vision IAS


Q 66.B
• In rocks like limestones or dolomites rich in calcium carbonate, the surface water as well
as groundwater through the chemical process of solution and precipitation deposition develop varieties
of landforms. These two processes of solution and precipitation are active in limestones or
dolomites occurring either exclusively or interbedded with other rocks. Any limestone or dolomitic region
showing typical landforms produced by the action of groundwater through the processes of solution and
deposition is called Karst topography.
• Limestone pavements are produced by the removal of surface material, and the vertical fissures along
joints are gradually widened and deepened, producing a grooved and jagged terrain. As it flows along
cracks underground, the water continues to widen and deepen the cracks until they become cave systems
or underground stream channels into which narrow vertical shafts may open. Most, but not all, of the
principal cave areas of the world are areas of karsts. Features such as lapiés, natural bridges, and Pepino
hills are characteristic of karsts.
• Some examples of Karst topography in India are
o The Kailash and Kotumsar caves: These are located in the Bastar region of Chhattisgarh: These
limestone caves extend deep underground inside Kanger Vally National Park.
o Belum Caves: The Belum Caves is the largest and longest cave system open to the public. It is known
for its speleothems, such as stalactite and stalagmite formations. These are located in Andhra
Pradesh.
o Borra caves: These caves are located in Andhra Pradesh. They exhibit a variety of speleothems
ranging in size and irregularly shaped stalactites and stalagmites.
• Krem Puri: The world's longest sandstone cave named Krem Puri was discovered in Meghalaya in
2016. It has a length of 24.5 km, and contains some dinosaur fossils from 66-76 million years ago as
well. Krem means 'cave' in Khasi language. It is the second-longest cave system in India after the Krem
Liat Prah-Umim-Labit limestone cave system located in Jaintia Hills.

Q 67.C
• Option 1 is not correct: Under Article 352, the President can declare a national emergency when the
security of India or a part of it is threatened by war or external aggression or armed rebellion. The
proclamation of Emergency must be approved by both the Houses of the Parliament within one month
from the date of its issue. Originally, the period allowed for approval by the Parliament was two months
but was reduced by the 44th Amendment Act of 1978. If approved by both the Houses of Parliament, the
emergency continues for six months and can be extended to an indefinite period with the approval of the
Parliament every six months.
• Option 2 is not correct: A proclamation imposing President’s Rule in a state must be approved by both
the Houses of Parliament within two months from the date of its issue. If approved by both the Houses of
Parliament, the President’s Rule continues for six months6. It can be extended for a maximum period of
three years7 with the approval of the Parliament, every six months.
• Option 3 is correct: Article 360 empowers the president to proclaim a Financial Emergency if he is
satisfied that a situation has arisen due to which the financial stability or credit of India or any part of its
territory is threatened. A proclamation declaring financial emergency must be approved by both the
Houses of Parliament within two months from the date of its issue. Once approved by both the Houses of
Parliament, the Financial Emergency continues indefinitely till it is revoked. This implies two things:
o there is no maximum period prescribed for its operation;
o and repeated parliamentary approval is not required for its continuation.

Q 68.D
• The Deep Carbon Observatory (DCO) is a global community of more than 1000 scientists on a ten-year
quest to understand the quantities, movements, forms, and origins of carbon in Earth. Hence, statement 1
is not correct.
• In 2009 a team of scientists launched an international collaboration, with the support of the Alfred P.
Sloan Foundation, to investigate how the deep carbon cycle drives our world. Hence, statement 2 is not
correct.
• DCO brings together a multidisciplinary group of scientists, including geologists, chemists, physicists,
and biologists. DCO scientists conduct research at approximately 100 field sites around the world.
• Using innovative technology and instrumentation, laboratory experiments, and real-time observations,
DCO scientists are answering the question of how deep carbon affects life on Earth.

28 www.visionias.in ©Vision IAS


• It recently founded that human activity churns out up to 100 times more planet-warming carbon each year
as all the volcanoes on Earth. Manmade emissions in 2018 alone topped 37 gigatonnes. Hence, both
statements are not correct.
• The Orbiting Carbon Observatory is a series of Earth satellite missions by NASA designed to study the
sources and sinks of carbon dioxide globally and provide scientists with a better idea of how carbon is
contributing to climate change.

Q 69.D
• The tradition of constructing pillars is very old and it may be observed that erection of pillars was
prevalent in the Achamenian empire as well. But the Mauryan pillars are different fromthe Achamenian
pillars.
• Statement 1 is not correct The Mauryan pillars are monolithic rock-cut pillars thus displaying the
carver’s skills, whereas the Achamenian pillars are constructed in pieces, by placing slabs of stoens onto
each other, by a mason.
• Statement 2 is correct: Monumental images of Yaksha, Yakhinis and animals, pillar columns with
capital figures, rock-cut caves belonging to the third century BCE have been found in different parts of
India. It shows the popularity of Yaksha worship and how it became part of figure representation in
Buddhist and Jaina religious monuments. Stone pillars were erected all over the Mauryan Empire with
inscriptions engraved on them.
• Statement 3 is correct: Ashoka issued edicts that carried his message concerning the idea and practice
of dhamma, the Prakrit form of the Sanskrit dharma. His inscriptions were deciphered in 1837 by British
archeologist and historian James Prinsep. After the edicts were deciphered, it was discovered that they
corroborated the assertions of the Buddhist sources, because in some of the edicts Ashoka avowed his
personal support of Buddhism. However, more-recent analyses suggest that, although he was personally
a Buddhist, as his edicts addressed to the Buddhist sangha attest, the majority of the edicts in which he
attempted to define dhamma do not suggest that he was merely preaching Buddhism.

Q 70.B
• Sinai Peninsula is located in the Asian part of Egypt between the Mediterranean sea to the north and the
Red Sea to the South. It is defined by the Suez canal and the Gulf of Suez in the west and the Gulf of
Aqaba in the southeast.
• Usually regarded as being geographically part of Asia, the Sinai Peninsula is the northeastern extremity of
Egypt and adjoins Israel and the Gaza Strip on the east. The Sinai is administratively divided into two
muḥafaẓahs (governorates): Shamāl Sinaʾ in the north and Janūb Sinaʾ in the south.

Q 71.D
• The IMF is a quota-based institution. Quotas are the building blocks of the IMF’s financial and
governance structure. An individual member country’s quota broadly reflects its relative position in the
world economy. Quotas are denominated in Special Drawing Rights (SDRs), the IMF’s unit of account.
29 www.visionias.in ©Vision IAS
• Multiple roles of quotas:
o Resource Contributions: Quotas determine the maximum amount of financial resources a member is
obliged to provide to the IMF.
o Voting Power: Quotas are a key determinant of the voting power in IMF decisions. Votes comprise
one vote per SDR100,000 of quota plus basic votes (same for all members).
o Access to Financing: The maximum amount of financing a member can obtain from the IMF under
normal access is based on its quota.
o SDR Allocations: Quotas determine a member’s share in a general allocation of SDRs.

Q 72.B
• Ryotwari System was a land revenue system introduced by the British in India. In this system ownership
rights were handed over to the peasants and British Government collected taxes directly from the
peasants.
• Statement 1 is correct: The first land revenue settlements were made in the Baramahal district of
Madras Presidency after its acquisition by the company in 1792 by Alexander Reed. Later it was
continued by Thomas Munro when he took charge of the revenue administration of the Ceded
Districts. Thomas Munro extended the ryotwari system to all parts of the province except the permanent
settlement areas. In Bombay Presidency too company decided in favor of the Ryotwari system with a
view to the elimination of landlord or village communities which could intercept their profits. It was also
introduced in Sind, Assam, and Coorg regions of India.
• Statement 2 is correct: The system required a detailed land survey. The quality of soil, the area of the
field and the average produce of every piece of land had to be assessed and on the basis of that the amount
of revenue was to be fixed. But this was the theory; in practice, the estimates were often guesswork and
the revenue demanded was often so high that they could only be collected with great difficulty or could
not be collected at all.
• Statement 3 is not correct: Under this system individual proprietary rights in land were vested in the
peasants, rather than in the zamindars. and responsible to direct payment of land revenue to the state. He
had right to sub-let his land holdings, to transfer, mortagage or sell it.

Q 73.C
• Minimum Export Price (MEP) is the price below which an exporter is not allowed to export the
commodity from India. MEP is imposed in view of the rising domestic retail / wholesale price or
production disruptions in the country. MEP is a kind of quantitative restriction to trade.
• The government fixes MEP for the selected commodities with a view to arrest domestic price rise and
augment domestic supply. This is intended to be imposed for short durations and is removed when
situations change.
• The removal of MEP helps farmers/exporters in realizing better and remunerative prices and would also
help in earning valuable foreign exchange for the country.
• MEP decisions are generally taken at the Cabinet-level. The requests for changes in MEP are brought to
the Cabinet by the line ministries, such as the Agriculture Ministry, who request changes generally on an
ad-hoc basis. It is not a formula based imposition.
• The Foreign Trade (Development And Regulation) Act, 1992, provides legal backing for MEP. As per
section 5 of this act, the Central Government may, from time to time, formulate and announce by
notification in the Official Gazette, the export, and import policy and may also, in the like manner, amend
that policy.

Q 74.D
• The IUCN Red List of Threatened Species is widely recognized as the most comprehensive, objective
global approach for evaluating the conservation status of plant and animal species. It is a loose-leaf
volume of information on the status of many kinds of species. It is not an annual magazine.
• In 1964 the first comprehensive list of threatened mammals and birds was compiled and
published. It enabled the general public access to the data.
• All birds were assessed for the first time in 1988. Some other species are assessed in1998 all Conifers, in
2004 all Amphibians, in 2008 all Mammals; Cycads and Reef Building Corals; in 2011 all Tuna and in
2012 all Sharks and Rays.
• IUCN Red Data Book provides scientifically correct information on the current status of globally
threatened biodiversity.

30 www.visionias.in ©Vision IAS


• Plants, fungi and animals that have been assessed and have a low risk of extinction are classified as Least
Concern.
• The Least Concern assessments did not appear on IUCN Red Lists produced before 2003 (except for a
few that were listed in 1996). Just for the sake of transparency, all Least Concern assessments are now
included on The IUCN Red List.
• The IUCN Red List are published periodically (usually at least once every four years).
• For each searched species, The IUCN Red List provides information on population size and trends;
geographic range and habitat needs. Till date more than 76,000 species have been searched with more
than 22,000 are at risk of extinction.

Q 75.B
• Ibn Battuta was a Moroccan scholar, and explorer who widely travelled the medieval world including
India. His book of travels, called Rihla, written in Arabic, provides extremely rich and interesting details
about the social and cultural life in the subcontinent in the fourteenth century.
• He observed that the state evidently took special measures to encourage merchants. Almost all trade
routes were well supplied with inns and guest houses.
• Ibn Battuta was amazed by the efficiency of the postal system which allowed merchants to not only
send information and remit credit across long distances but also to dispatch goods required at short
notice. The postal system was so efficient that while it took fifty days to reach Delhi from Sind, the news
reports of spies would reach the Sultan through the postal system in just five days. Hence statement 1 is
not correct.
• Ibn Battuta found cities were full of exciting opportunities for those who had the necessary drive,
resources, and skills. The bazaars in these cities were not only places of economic transactions, but
also the hub of social and cultural activities. Most bazaars had a mosque and a temple, and in some of
them at least, spaces were marked for public performances by dancers, musicians and singers. Hence
statement 2 is correct.
• As per the accounts of Ibn Battuta’s, there was considerable differentiation among slaves. Some female
slaves in the service of the Sultan were experts in music and dance. Female slaves were also
employed by the Sultan to keep a watch on his nobles. Slaves were generally used for domestic labour,
and Ibn Battuta found their services particularly indispensable for carrying women and men on palanquins
or dola. The price of slaves, particularly female slaves required for domestic labour, was very low, and
most families who could afford to do so kept at least one or two of them. Hence statement 3 is not
correct.

Q 76.A
Important Glaciers of the Himalayas Mountains:
Name of Glacier Location
• Zemu Kanchenjunga-Everest
• Satopanth Glacier Kumaon-Garhwal
• Sonapani Pir Panjal
• Hispar Karakoram
• Biafo Karakoram
• Baltoro Karakoram
• Rongbuk Kanchenjunga-Everest
• Chongo-Lungma Karakoram
• Khurdaplo Karakoram
• LoLofond Karakoram
• Yarkand Rimo Karakoram
• Gangotri Kumaon-Garhwal
• Godwin Austen Karakoram
• Pasu Karakoram
• Siachin Eastern Karakoram range in the Himalayas
• Chong Kumdan Karakoram
• Kanchenjunga Kanchenjunga-Everest
• Milam Kumaon-Garhwal
• Chungpur Pir Panjal
• To Lam Bau Kanchenjunga-Everest

31 www.visionias.in ©Vision IAS


• Bhagirath Kharak Kumaon-Garhwal
• Fedchenko Karakoram
• Bara Shighi Pir Panjal
• Rakhiot Pir Panjal
• Gangri Pir Panjal
• Rambang Kanchenjunga-Everest
• Kafni Glacier Kumaon-Garhwal
• Kalabaland Glacier Kumaon-Garhwal
• Kedar Bamak Glacier Kumaon-Garhwal
• Meola Glacier Kumaon-Garhwal
• Namik Glacier Kumaon-Garhwal
• Panchchuli Glacier Kumaon-Garhwal
• Pindari Glacier Kumaon-Garhwal
• Ralam Glacier Kumaon-Garhwal
• Sona Glacier Kumaon-Garhwal
• Sunderdhunga Glacier Kumaon-Garhwal
• Dokriani Glacier Kumaon-Garhwal
• Chorabari Glacier Kumaon-Garhwal
• Lonak North-east Himalayas
• Chhota Shigri Pir Panjal
• Trango Karakoram

Q 77.D
• Labour force refers to those who are either engaged in any economic activities or are willing to pursue an
economic activity in a reference period. It includes both (i) those who are in the workforce; and
(ii) unemployed.
• Of these, workforce refers to the population who are actively engaged in any economic activities and
producing goods and services in a reference period while unemployed refers to all those who are
seeking and available for work but had not worked in a reference year due to lack of work.
• Hence, Labour force participation rate (LFPR) can be defined as the proportion of the population in the
labour force to the total population.
• Worker population ratio (WPR) can be defined as the proportion of employed persons to the total
population.

Q 78.C
• The judges of a high court are appointed by the President. The chief justice is appointed by the President
after consultation with the Chief Justice of India and the governor of the state concerned. As per the Third
Judges case (1998), the sole opinion of the Chief Justice of India alone does not constitute the
‘consultation’ process. Thus the Chief Justice of India should consult a collegium of two senior-most
judges of the Supreme Court.
• For the appointment of other judges, the chief justice of the concerned high court is also consulted. A
judge of a high court can be removed from his office by an order of the President. The President can issue
the removal order only after an address by the Parliament has been presented to him in the same session
for such removal. In the removal of a high court judge, the collegium plays no role. Hence option (a) is
not correct.
• The President can transfer a judge from one high court to another after consulting the Chief Justice of
India. On transfer, he is entitled to receive in addition to his salary such compensatory allowance as may
be determined by Parliament. In 1977, the Supreme Court ruled that the transfer of high court judges
could be resorted to only as an exceptional measure and only in the public interest and not by way of
punishment. Hence option (b) is not correct.
• Again in 1994, the Supreme Court held that judicial review is necessary to check arbitrariness in the
transfer of judges. But, only the judge who is transferred can challenge it. Hence option (c) is the correct
answer.
• In the Third Judges case (1998), the Supreme Court opined that in case of the transfer of high court
judges, the Chief Justice of India should consult, in addition to the collegium of four seniormost judges of
the Supreme Court, the chief justice of the two high courts (one from which the judge is being transferred
and the other receiving him). Thus, the sole opinion of the Chief Justice of India does not constitute the
‘consultation’ process.

32 www.visionias.in ©Vision IAS


• The retired permanent judges of a high court are prohibited from pleading or acting in any court or before
any authority in India except the Supreme Court and the other high courts (that does not include his own
high court). This ensures that they do not favour anyone in the hope of future favour. Hence option (d) is
not correct.

Q 79.C
• According to the older conceptions, the Buddha wrought many deeds of kindness and mercy in a long
series of transmigrations as a Bodhisattva. The Jataka stories show that Bodhisattvas can be incarnated as
men, or even as animals; but the more advanced Bodhisattvas, who have the greatest power for good,
must be divine beings in the heavens. Bodhisattvas existing at present in the universe are working
continuously for the welfare of all things living.
• The universe of the Great Vehicle(Mahayana) contains numerous Bodhisattvas, chief of whom, from the
earthly point of view, is Avalokitesvara ("The Lord Who Looks Down"), also called Padmapani
("The Lotus-Bearer") also called Lokesvara in Sanskrit ( The Lord of the World). His special
attribute is compassion, and his helping hand reaches even to Avici, the deepest and most unpleasant of
the Buddhist purgatories. The Avalokitesvara Padmapani is painted in cave 1 of Ajanta. In the painting, he
sits with a calm, spiritual face holding a lotus blossom, which is seen as his spiritual awakening. Hence
option (c) is the correct answer.
• Another important Bodhisattva is Manjusri, whose special activity is to stimulate the understanding,
and who is depicted with a naked sword in one hand, to destroy error and falsehood, and a book in the
other, describing the ten paramitas, or great spiritual perfections, which are the cardinal virtues developed
by Bodhisattvas.
• Vajrapani, a sterner Bodhisattva, is the foe of sin and evil, and, like the god Indra, bears a
thunderbolt in his hand.
• The gentle Maitreya, the future Buddha, is worshipped as a Bodhisattva.
• Also worthy of mention is Ksitigarbha, the guardian of the purgatories, who is thought of not as a fierce
torturer, but rather as the governor of a model prison, doing his best to make life tolerable for his charges,
and helping them to earn remission of sentence.

Q 80.C
• Adopted by the First International Conference on Chemicals Management (ICCM1) in February 2006 in
Dubai, the Strategic Approach to International Chemicals Management (SAICM) is a policy
framework to promote chemical safety around the world.
• SAICM was developed by a multi-stakeholder and multi-sectoral Preparatory Committee and supports the
achievement of the 2020 goal agreed at the 2002 Johannesburg World Summit on Sustainable
Development. SAICM's overall objective is the achievement of the sound management of chemicals
throughout their life cycle so that by the year 2020, chemicals are produced and used in ways that
minimize significant adverse impacts on the environment and human health. It is a non-binding policy
framework. The SAICM Secretariat is hosted by the United Nations Environment Programme.
Hence statement 1 is not correct and statement 2 is correct.
• The Strategic Approach has a scope that includes:
o Environmental, economic, social, health and labour aspects of chemical safety
o Agricultural and industrial chemicals, with a view to promoting sustainable development and covering
chemicals at all stages of their life-cycle, including in products.
• In February 2006, over 190 countries including India acceded to the Strategic Approach to
International Chemicals Management (SAICM). Initial activities under SAICM included development
or updating of national chemicals profiles, strengthening of institutions, and mainstreaming sound
management of chemicals in national strategies. Towards this end, India initiated the preparation of the
National Chemicals Management Profile to assess India’s infrastructure and capacity for the management
of chemicals. Hence statement 3 is not correct.

Q 81.B
• Conditions favourable for tropical cyclones:
o Sea surface temperature (26.5 degrees)
o Sufficient Coriolis force
o Low-level positive vorticity
o Weak vertical wind shear of horizontal winds
o Large convective instability

33 www.visionias.in ©Vision IAS


o Low-level convergence of the winds
o Huge supply of moisture from the sea surface.
• Among them everything has to be satisfied, then only the cyclone will form. But, near the equatorial
region, all conditions are almost available excluding Coriolis force. The rotation of the earth about its
axis affects the direction of the wind and this force is called the Coriolis force. It deflects the wind to the
left direction in the southern hemisphere and the right direction in the northern hemisphere.
• In low-pressure areas, within the tropics, when the Coriolis force acts perpendicular to the pressure
gradient force (change of pressure with respect to distance), the wind blows around it and gets intensified
to form tropical cyclones.
• But, at the equator, the angle of latitude is zero, so the Coriolis force is also zero. As there is no
Coriolis force to deflect, the winds blow perpendicular to the isobars and the low pressure gets filled
directly.
• There is no scope for cyclical movement of the wind, so no intensification of the accumulated winds.
Hence there cannot be a tropical cyclone at the equator although all other ingredients are present.

Q 82.B
• Scientists from Technion — Israel Institute of Technology have developed a concept they have named the
Ramanujan Machine, after the Indian mathematician. It is not really a machine but an algorithm and
performs a very unconventional function. It is a device that automatically generates conjectures
(mathematical statements that are proposed as true statements) for fundamental constants.
• With most computer programs, humans input a problem and expect the algorithm to work out a solution.
With the Ramanujan Machine, it works the other way round. Feed-in a constant, say the well-know pi,
and the algorithm will come up with an equation involving an infinite series whose value, it will propose,
is exactly pi. Over to humans now: let someone prove that this proposed equation is correct.
• Hence option (b) is the correct answer.

Q 83.C
• Marginal Standing Facility (MSF) is a new scheme announced by the Reserve Bank of India (RBI) in its
Monetary Policy (2011-12). It refers to the penal rate at which banks can borrow money from the central
bank over and above what is available to them through the LAF window.
• It is a window for banks to borrow from the Reserve Bank of India in an emergency situation when inter-
bank liquidity dries up completely.
• The MSF would be the last resort for banks once they exhaust all borrowing options including the
liquidity adjustment facility by pledging government securities, where the rates are lower in comparison
with the MSF. Hence, statement 2 is correct.
• The MSF would be a penal rate for banks and the banks can borrow funds by pledging government
securities within the limits of the statutory liquidity ratio. MSF, being a penal rate, is always fixed above
the repo rate. The MSF rate is usually pegged 100 basis points or a percentage point above the repo
rate Hence, statement 1 is correct.
• The scheme has been introduced by RBI with the main aim of reducing volatility in the overnight lending
rates in the inter-bank market and to enable a smooth monetary transmission in the financial system.

Q 84.C
• The Communist Party of India organized the workers' and peasants' on class lines. The main form of
political work by the early Communists was to organize peasants’ and workers’ parties and work through
them.
• The first such organization was the Labour-Swaraj Party of the Indian National Congress organized
by Muzaffar Ahmed, Qazi Nazrul Islam, Hemanta Kumar Sarkar, and others in Bengal in November
1925. In late 1926, a Congress Labour Party was formed in Bombay and a Kirti-Kisan Party in
Punjab.
• A Labour Kisan Party of Hindustan had been functioning in Madras since 1923.
• By 1928 all of these provincial organizations had been renamed the Workers’ and Peasants’ Party
(WPP) and knit into an All India party, whose units were also set up in Rajasthan, UP and Delhi.
• The primary objective of the WPPs was to work within the Congress to give it a more radical
orientation and make it ‘the party of the people’ and independently organize workers and peasants
in class organizations, to enable first the achievement of complete independence and ultimately of
socialism. Hence, statement 1 is correct.
• All Communists were members of the All India Workers and Peasant party.

34 www.visionias.in ©Vision IAS


• The WPPs grew rapidly and within a short period, the communist influence in Congress began to grow
rapidly, especially in Bombay. Moreover, Jawaharlal Nehru and other radical Congressmen welcomed the
WPPs’ efforts to radicalize the Congress. Along with Jawaharlal and Subhas Bose, the youth leagues
and other Left forces, the WPPs played an important role in creating a strong left-wing within the
Congress and in giving the Indian national movement a leftward direction. Hence statement 2 is
correct.
• Later, the Workers’ and Peasants’ Party was also dissolved on the ground that it was inadvisable to form a
two-class (workers’ and peasants’) party for it was likely to fall prey to petty-bourgeois influences.

Q 85.B
• The Constitution does not grant any immunity to the ministers for their official acts. But, since they
are not required to countersign (as in Britain) the official acts of the president and the governors, they are
not liable in the courts for those acts. Moreover, they are not liable for the official acts done by the
president and the governors on their advice as the courts are debarred from inquiring into such advice.
However, the ministers do not enjoy any immunity for their personal acts and can be sued for crimes as
well as torts (wrongful acts) in ordinary courts like common citizens. Hence statement 1 is correct and
statement 2 is not correct.
• Under the Constitution, the civil servants are conferred personal immunity from legal liability for official
contracts. This means that the civil servant who made a contract in his official capacity is not personally
liable in respect of that contract but it is the government (Central or state) that is liable for the contract.
But, if the contract is made without complying with the conditions specified in the Constitution, then the
civil servant who made the contract personally liable. Hence statement 3 is correct.
• According to Article 310, members of the defense services (central services), the civil services of the
Centre and the all-India services or persons holding military posts or civil posts under the Centre, hold
office during the pleasure of the president. Similarly, members of the civil services of a state (who are
appointed by the governor of that state) or persons holding civil posts under a state, hold office during the
pleasure of the governor of the state. Hence statement 4 is not correct.
Q 86.A
• Manipravalam was a common liturgical language during the medieval period in Kerala and is basically a
mixture of Sanskrit and Tamil. In medieval Kerala, around the 11th, 12th and 13th centuries, the elite,
especially the Namboodiri Brahmins, crafted a language that blended early Malayalam with Tamil
and Sanskrit to fashion a distinct literary tongue referred to as Manipravalam. The possible reason
as to why such a tongue was fashioned probably has something to do with Namboodiri ascendancy in
Kerala and their view of Sanskrit as “high" culture, as opposed to the vernacular that was used by the
ordinary folk.
• The word mani meant “ruby" in Malayalam; pravalam meant “coral" in Sanskrit. According to
many historians, Manipravalam is seen as origin to modern Malayalam.
• Important works in Manipravalam:
• In the 12th or 13th-century, Vaiśikatantram was probably the first work in Manipravalam. It was
manual composed in the didactic mode on the art and tradition of the courtesan, this work discusses the
profession of vaiśikavritti (the courtesan’s work). In this work, a mother instructs the daughter in the finer
details of the courtesan (Devadasi) tradition.
• Lilatilakam was a 14th-century treatise of uncertain authorship discussed the finer points of
grammar and poetics.
• Many Manipravalam poems (like its first work, Vaiśikatantram) were also composed in the erotic
tradition e.g The Chandrotsavam (Moon Festival, written around 1500 AD) is a satire on this aspect of
the tradition.
• Besides poetry, Manipravalam was also used to write scientific treatises on subjects like astronomy
and medicine.
• Though Koodiyattam pre-dates Manipravalam, but recently Kathakali and Koodiyattam have
found influence from Manipravalam.

Q 87.C
• Money markets are used for short-term lending or borrowing usually the assets are held for one year
or less whereas, Capital Markets are used for long-term securities they have a direct or indirect impact
on the capital.
• Capital markets are composed of primary and secondary markets. The most common capital markets
are the stock market and the bond market.

35 www.visionias.in ©Vision IAS


• The Securities and Exchange Board of India Act, 1992 (SEBI Act, 1992) established Securities and
Exchange Board of India (SEBI) primarily to protect the interests of investors in securities and to promote
the development of, and to regulate, the securities market i.e. Capital Market.
• In the Budget Speech of 2015, the merger of the Forwards Markets Commission (FMC) with
SEBI was announced. Prior to that, FMC had the regulatory oversight over the commodity derivatives
markets under Forward Contracts (Regulation) Act, 1952 (“FCRA”). Through the Finance Act, 2015,
FCRA, was repealed. Consequently, all recognized associations (i.e. commodity derivatives exchanges)
under FCRA have been deemed to be recognized stock exchanges under the Securities Contracts
(Regulation) Act, 1956 (“SCRA”) and SEBI started regulating the commodity derivatives market.
Hence option (c) is the correct answer.
• The Reserve Bank regulates money markets, Government Securities (G-Sec) market, foreign
exchange (Forex) market and the markets for derivatives on interest rate, currency and credit
derivatives.
o Forex market in India is predominantly a wholesale market, dominated by banks, forex brokers and
corporate clients. Customers are priced off-market by banks. Trading in forex and related derivatives
takes place OTC as well as on exchanges.
o Money Market: Overnight money market in India includes the call money market, tri-party repos
(TREPS) and market repos.

Q 88.D
• Tebhaga literally means 'three shares' of harvests. Traditionally, sharecroppers used to hold their tenancy
on a fifty-fifty basis of the share of the produce. Tebhaga Movement was the sharecroppers' (bargadars)
movement in Bengal during 1946, demanding two-thirds of the produce from the land for themselves
and one third for the landlords. Hence, statement (a) is correct. They also demanded that before
division the crop would be stored in their khamars (godowns) and not that of the jotedars.
• The Tebhaga movement, led by the Bengal Provincial Kisan Sabha, thus soon developed into a clash
between jotedars and bargadars.
• The main centers of the movement were Dinajpur, Rangpur, Jalpaiguri, Mymensingh, Midnapore, and to a
lesser extent 24- Parganas and Khulna. Initially, the base was among the Rajbansi Kshatriya peasants,
but it soon spread to Muslims and tribals like Hajongs, Santhals and Oraons. Among the important
leaders of this movement were Krishnobinode Ray, Abani Lahiri, Sunil Sen, Bhowani Sen, Moni Singh,
Ananta Singh, Bhibuti Guha, Ajit Ray, Sushil Sen, Samar Ganguli, and Gurudas Talukdar. Hence
statement (b) is correct.
• Among the unique features of the Tebhaga movement was the large-scale participation of women on par
with men. The landless and poor peasant women formed fighting troops called Nari Bahini and took
a front rank role in defending the gains of the movement and in countering the repression of the
state. Hence statement (c) is correct.
• The movement was encouraged by the fact that the Bengal Land Revenue Commission, popularly known
as the Floud Commission, had already made this recommendation in its report to the government. Thus,
Floud Commission was not constituted to look into demand of the Tebhaga movement. Hence
statement (d) is not correct.
• The movement received a great boost in late January 1947 when the Muslim League Ministry led by
Suhrawardy published the Bengal Bargadars Temporary Regulation Bill in the Calcutta Gazette on 22
January 1947. Encouraged by the fact that the demand for Tebhaga could no longer be called illegal,
peasants in hitherto untouched villages and areas joined the struggle. The Muslim League Ministry failed
to pursue the bill in the Assembly and it was only in 1950 that the Congress Ministry passed a Bargadars
Bill which incorporated, in substance, the demands of the movement.

Q 89.B
• Statement 1 is not correct: A Participatory Note (PN or P-Note) in the Indian context, in essence, is a
derivative instrument issued in foreign jurisdictions, by a SEBI registered Foreign Institutional Investor
(FII) or its sub-accounts or one of its associates, against underlying Indian securities. The underlying
Indian security instrument may be equity, debt, derivatives or may even be an index. Further, a basket of
securities from different jurisdictions can also be constructed in which a portion of the underlying
securities is Indian securities or indices.
• PNs are market instruments that are created and traded overseas. Hence, Indian regulators cannot ban the
issue of PNs. However, they can only be regulated, and they are being regulated by the securities market

36 www.visionias.in ©Vision IAS


regulator in India, Securities, and Exchange Board of India (SEBI). When a PN is traded on an overseas
exchange, the regulator in that jurisdiction would be the authority to regulate that trade.
• Statement 2 is correct: The investor in PN does not own the underlying Indian security, which is held by
the FII who issues the PN. Thus the investors in PNs derive the economic benefits of investing in the
security without actually holding it. They benefit from fluctuations in the price of the underlying security
since the value of the PN is linked with the value of the underlying Indian security. The PN holder also
does not enjoy any voting rights in relation to security/shares referenced by the PN.
Q 90.B
Both the CVC (Central Vigilance Commission) and the CIC (Central Information Commission) are statutory
bodies. In 2003, the Parliament enacted a law conferring statutory status on the CVC, while CIC was
constituted in the year 2005 under the Right to Information Act of 2005. Hence option 1 is correct.
The differences between the two bodies are as given below.
CVC
• The CVC is a multi-member body consisting of a Central Vigilance Commissioner (chairperson) and not
more than two vigilance commissioners. They are appointed by the president by warrant under his hand
and seal on the recommendation of a three-member committee consisting of the prime minister as its
head, the Union minister of home affairs and the Leader of the opposition in the Lok Sabha.
• They hold office for a term of four years or until they attain the age of sixty-five years, whichever is
earlier. After their tenure, they are not eligible for further employment under the Central or a state
government.
• The salary, allowances and other conditions of service of the Central Vigilance Commissioner are
similar to those of the Chairman of UPSC and that of the vigilance commissioner are similar to those of
a member of UPSC.
CIC (Before the Right to Information Amendment Act 2019)
• CIC was constituted in the year 2005 under the Right to Information Act, 2005. The Commission consists
of a Chief Information Commissioner and not more than ten Information Commissioners. They are
appointed by the President on the recommendation of a committee consisting of the Prime Minister as
Chairperson, the Leader of Opposition in the Lok Sabha and a Union Cabinet Minister nominated
by the Prime Minister.
• The Chief Information Commissioner and an Information Commissioner hold office for a term of 5
years or until they attain the age of 65 years, whichever is earlier. They are not eligible for reappointment.
• The salary, allowances and other service conditions of the Chief Information Commissioner are similar
to those of the Chief Election Commissioner and that of the Information Commissioner are similar to
those of an Election Commissioner. But, they cannot be varied to his disadvantage during service.
CIC (After the Right to Information Amendment Act 2019)
• The RTI Amendment Bill, 2019 (now RTI Amendment Act 2019) amends Sections 13 and 16 of the Right
to Information (RTI) Act, 2005.
• Section 13 of the original Act sets the term of the central Chief Information Commissioner and
Information Commissioners at five years (or until the age of 65, whichever is earlier). The amendment
proposes that the appointment will be “for such term as may be prescribed by the Central
Government”.
• Again, Section 13 states that salaries, allowances and other terms of service of “the Chief Information
Commissioner shall be the same as that of the Chief Election Commissioner”, and those of an Information
Commissioner “shall be the same as that of an Election Commissioner”. The amendment proposes that the
salaries, allowances and other terms of service of the Chief Information Commissioner and the
Information Commissioners “shall be such as may be prescribed by the Central Government”.
• Hence options 2 and 3 are not correct.

Q 91.A
• Xerosere is a plant succession that is limited by water availability. It includes the different stages in a
xerarch succession. Xerarch succession of ecological communities originated in extremely dry situations
such as sand deserts, sand dunes, salt deserts, rock deserts etc.
• The pioneers to colonise this primitive substratum are crustose type of lichens, and through a series of
successive serai stages, the succession finally terminates into a forest which constitutes the climax
community.
37 www.visionias.in ©Vision IAS
• The various stages in xerosere are:
o Lichen stage: Due to great exposure to sun and extreme deficiency of water, the first pioneers on
the bare rock area are a few simple organisms. The most successful of such organisms are crustose
lichens. These are able to withstand extreme desiccation due to excessive dryness. The common
species of crustose lichens are Rhizocarpon, Rinsodina etc.
o Moss stage: With the accumulation of dust and humus in small quantities the environment is altered
enough to allow the establishment of secondary communities in a rather definite sequence. Mosses
like Funaria, Sphagnum and Polytrichum make their appearance.
o Herbaceous stage: As the mats of mosses become more extensive, more soil accumulates; much of
the soil is blown in from the surrounding area during windy periods. More mineral material is added
to the soil as acid leaches out from the overlying vegetation and increases the depth of the mineral soil
layer. Many annual weeds develop which are, later on, followed by biennial and finally perennial
grasses. With the influx of grasses, the fauna (animals) also becomes varied. Nematodes and larval
insects appear gradually.
o Shrub stage: Further modification of the environment provides conditions for the germination and
growth of shrubs and perennial wood plants such as Acacia, Prosopis, Capparis, Zizyphus etc.
o Climax forest: With the establishment of shrubs, more and more soil is formed and the environment
becomes increasingly humid. This favours the growth of woody trees. In the beginning, trees show
stunted growth and are sparsely placed. Finally, a climax forest community is established.
• Correct sequence is Lichen-Moss-Herb-Shrub.

Q 92.B
• Statement 1 is correct: Transfer pricing refers to the rules and methods for pricing transactions within
and between enterprises under common ownership or control.
• Statement 3 is not correct: According to the Indian Income-tax Act, 1961, income arising from such
transactions must be computed using the arm’s length price principle, that is, the amount payable if the
trading companies were unrelated or uncontrolled. A separate code on transfer pricing under Sections 92
to 92F of the Indian Income TaxAct, 1961 (the Act) covers intra-group cross-border transactions. The Act
defines the terms‘international transactions’, ‘specified domestic transactions’, ‘associated enterprises’
and ‘arm’s-length price’.
• Statement 2 is correct: Transfer pricing can be exploited for tax avoidance and tax evasion through Base
erosion and profit shifting strategies.
• Base erosion and profit shifting refer to corporate tax planning strategies used by multinationals to
"shift" profits from higher-tax jurisdictions to lower-tax jurisdictions, thus "eroding" the "tax-base" of the
higher-tax jurisdictions. The OECD, G20 and several other international organizations perceived BEPS a
major issue in international taxation.
• OECD published its first report on BEPS, in which transfer pricing regulation was named as a crucial tool
in the battle against BEPS
• It is aimed to “assure that transfer pricing outcomes are in line with value creation.”
• The background of these actions is that the view of the G20/OECD was that existing transfer pricing
guidance could result in outcomes that did not align operational profits with the economic activities which
generate them.

Q 93.D
• Anasuya Sarabhai was born in 1885 into a business family of Ahmedabad. She pioneered women's
labor movement in India. She founded the Ahmedabad Textile Labour Association (Majoor
Mahajan Sangh), India’s oldest union of textile workers, in 1920. In 1912 she went to London for
higher studies where she got influenced by Fabian society and suffragette movement.
• In 1913, she returned to India and started working with the disempowered communities. She began by
opening a school for poor students of all castes, whom she would bathe and teach herself. Next, she
opened creches and toilets for women, a maternity home and even a hostel for harijan girls in her home.
• Anasuya took up the task to change the situation of mill workers. In 1914 when an epidemic hit the
Ahemdabad the condition of the mill workers deteriorated further and they approached Anasuya to take
up their cause. She gave an ultimatum to the mill owners and even went against her brother, Ambalal,
who was the then-president of the Mill Owners’ Association, demanding better wages and working
ambiance for the laborers. Her endeavor was successful and the trade union movement in India took its
baby steps.

38 www.visionias.in ©Vision IAS


• She was supported in her work by Mahatma Gandhi and in 1918, Anasuya managed mill owners to accede
to the demand of Ahmedabad weavers for a 35% wage hike. Tens of thousands of workers participated in
the protest, laying the foundation for Gujarat’s oldest labour union, Majoor Mahajan Sangh (Ahmedabad
Textile Labour Association or TLA), that was established on February 25, 1920.
• Hence option (d) is the correct answer.

Q 94.C
• Indian scientists have discovered the presence of a sub-species of hog deer, Axis porcinus
annamiticus, in India. This endangered sub-species was earlier believed to be confined to the
eastern part of central Thailand. Researchers at Wildlife Institute of India (WII), Dehradun have
reported the presence of a small population of hog deer in Keibul Lamjao National Park (KLNP) in
Manipur. The population genetically resembles A. p. annamiticus.
• The Keibul Lamjao National is considered as a biodiversity hotspot on the India-Myanmar border. This
study indicates that the western limit of hog deer is Manipur and not central Thailand as believed so far.
Since the hog deer is losing its habitat in other countries, the genetically distinct and evolutionarily
significant population found in KLNP is significant for the conservation of the species.
• Two sub-species of hog deer have been reported from its range. The western race is distributed from
Pakistan and the Terai grasslands (along the Himalayan foothills, from Punjab to Arunachal Pradesh),
while the eastern race of hog deer is found in Thailand, Indo-China, Laos, Cambodia, and Vietnam. The
hog deer or 'Pada' is an endangered species in the IUCN Red List and is protected under Schedule I of
the Indian Wild Life (Protection) Act, 1972. Recently, a man in Assam has been sentenced to seven years
of imprisonment for killing a hog deer (Axis pornicus).

Q 95.B
• Trade winds: It is the wind blowing steadily towards the equator from the north-east in the northern
hemisphere or the south-east in the southern hemisphere, especially at sea. Two belts of trade winds
encircle the earth. These winds are called trade winds because of the fact that they helped the sea
merchants in sailing their ships as their (of trade winds) direction remain more or less constant and
regular. There is more or less regular inflow of winds from subtropical high-pressure belts to the
equatorial low-pressure belt. Hence statement 2 is correct.
• According to Ferrel's law (based on Coriolis force generated by the rotation of the earth), trade winds are
deflected to the right in the northern hemisphere and to the left in the southern hemisphere. Hence
statement 3 is correct.
• There is much variation in the weather condition in different parts of the trade winds. The poleward side
of the trade winds or eastern sides of the subtropical anticyclones are dry because of the strong subsidence
of air current from above.
• Because of the dominance of the anticyclonic conditions, there is strong atmospheric stability, strong
inversion of temperature and clear sky.
• On the other hand, the equatorward part of the trade winds are humid because they are characterized by
atmospheric instability and much precipitation as the trade winds while blowing over the oceans pick up
moisture.
• Trade winds are more regular and constant over the oceans than over the lands. At some places on the
lands (eg. S.E. Asia and the southern USA) the trade winds disappear during the summer season due to
the formation of low-pressure cells because of high temperature but the trade winds are more constant
and regular over the continents during the winter season. Hence statement 1 is not correct
• It may be pointed out that the zone of trade winds is called Hadley Cell on the basis of the convective
model prepared by Hadley for the entire earth.

Q 96.B
• Pair 1 is correctly matched: Gulamgiri is written by Jyotirao Phule. Phule’s works, Gulamgiri and,
Sarvajanik Satyadharma became sources of inspiration for the common masses. Jyotiba Phule (1827-
1890), born in Satara, Maharashtra, belonged to the mali (gardener) community. Phule was a prominent
social reformer and thinker of the nineteenth century India. He led the movement against the prevailing
caste-restrictions in India and organized a powerful movement against upper caste domination and
brahminical supremacy. Phule founded the Satyashodhak Samaj (Truth Seekers’ Society) in 1873, with
the leadership of the samaj coming from the backward classes, malis, telis, Kunis, saris and dhangars.The
main aims of the movement were (i) social service, and (ii) spread of education among women and lower
caste people.

39 www.visionias.in ©Vision IAS


• Pair 2 is not correctly matched: The book Stripurushtulna was written by Tarabai Shinde. She was a
women’s right’s activist who protested patriarchy and Caste system in 19th century India. In her
book Stripurushtulna, (A Comparison between Women and Men), she criticized the social differences
between men and women. She was also a member of Satyashodhak Samaj. Pandita Ramabai was a great
scholar of Sanskrit. She felt that Hinduism was oppressive towards women, and wrote a book about the
miserable lives of upper-caste Hindu women. She founded a widows’ home at Poona to provide shelter to
widows who had been treated badly by their husbands’ relatives.
• Pair 3 is correctly matched: Bahuvivah is a book by Ishwar Chandra Vidyasagar. He was a great
scholar and reformer and believed in high moral values. In 1850, he became the principal of Sanskrit
College. He was determined to break the priestly monopoly of scriptural knowledge, and for this, he
opened the Sanskrit College to non-brahmins. He introduced Western thought in Sanskrit College to break
the self-imposed isolation of Sanskritic learning. As an academician, he evolved a new methodology to
teach Sanskrit. He also devised a new Bengali primer and evolved a new prose style. Vidyasagar started a
movement in support of widow remarriage which resulted in the legalisation of widow remarriage. He
was also one of the pioneers of higher education for women in India.

Q 97.C
• Articles 301 to 307 in Part XIII of the Constitution deal with the trade, commerce, and intercourse within
the territory of India. Article 301 declares that trade, commerce, and intercourse throughout the territory
of India shall be free. The object of this provision is to break down the border barriers between the states
and to create one unit with a view to encouraging the free flow of trade, commerce, and intercourse in the
country.
• The freedom guaranteed by Article 301 is a freedom from all restrictions, except those which are provided
for in the other provisions (Articles 302 to 305) of Part XIII of the Constitution itself. These are explained
below:
o Parliament can impose restrictions on the freedom of trade, commerce, and intercourse between the
states or within a state in the public interest. But the Parliament cannot give preference to one state
over another or discriminate between the states except in the case of scarcity of goods in any part of
India (Article 303 (2)). Hence statement 1 is not correct.
o The legislature of a state can make a law or laws to impose reasonable restrictions on the freedom of
trade, commerce, and intercourse with that state or within that state in the public interest. But, a bill
for this purpose can be introduced in the legislature only with the previous sanction of the president.
Further, the state legislature cannot give preference to one state over another or discriminate between
the states. Hence statement 2 is not correct.
o The legislature of a state can impose on goods imported from other states or the union territories any
tax to which similar goods manufactured in that state are subject. This provision prohibits the
imposition of discriminatory taxes by the state. Hence statement 3 is correct.
o The freedom (under Article 301) is subject to the nationalization laws (i.e., laws providing for
monopolies in favor of the Centre or the states). Thus the Parliament or the state legislature can make
laws for the carrying on by the respective government of any trade, business, industry or service,
whether to the exclusion, complete or partial, of citizens or otherwise.
Q 98.C
• The Neora Valley National Park (NVNP) is situated in the Kalimpong district, West Bengal, India
and was established in 1986. The NVNP is a compact patch of virgin forest, rich in biodiversity located in
the Eastern Himalayas, a global 'biodiversity hotspot'. It was notified as a National Park based on the
provisions of the Wildlife (Protection) Act 1972 in the year 1992.
• The Kaas Plateau, also known as the Kaas Pathar, is a plateau is located in Maharashtra, India. It
falls under the Sahyadri Sub Cluster of the Western Ghats, and it became a part of a UNESCO World
Natural Heritage Site in 2012. It is a biodiversity hotspot known for various types of seasonal wild
flowers bloom and numerous species of endemic butterflies annually in the months of August and
September.
• Dudhsagar Falls is a four-tiered waterfall located on the Mandovi River in Goa. The falls is located in
the Bhagwan Mahaveer Sanctuary and Mollem National Park among the Western Ghats. The waterfall
forms the border between Karnataka and Goa states.
• Top Slip is the main tourist area of the Indira Gandhi Wildlife Sanctuary which lies in the Western
Ghats south of Palakkad Gap. The sanctuary is located in the Coimbatore district of Tamil Nadu. It is
also known as Grass Hill National Park. The Indira Gandhi Wildlife Sanctuary forms part of a larger

40 www.visionias.in ©Vision IAS


conservation area and runs contiguous with the Parambikulam Wildlife Sanctuary (to its west), and the
Eravikulam National Park and the Chinnar Wildlife Sanctuary to its south.
• Hence option (c) is the correct answer.

Q 99.A
• RBI has mandated all banks to link their floating rate loans to an external benchmark instead of the
marginal cost-based lending rate (MCLR). This was done to make sure that the RBI's action on key policy
rates is transmitted in a timely and transparent manner to the borrowers.
• The problem with the MCLR regime was that when RBI cut the repo and reverse repo rates, banks did not
pass the full benefits to borrowers.
Internal benchmarking of loan price, policy rate cuts often don't reach the borrowers. For example, RBI
has cut the key policy rates by a total of 135 basis points during 2019 but SBI has passed on the benefit of
65 bps in the case of MCLR.
• Under the new system, banks will have to link their lending rates with an external benchmark instead of
MCLR. One of these benchmarks will be used to decide the lending rate in addition to the spread, Banks
will be free to decide their spread value but it will have to be fixed for the tenure of the loan. Thus, the
interest rates are directly influenced by the policy rates provides effective transmission of monetary
policy. Hence, statement 1 is correct.
• The Reserve Bank of India (RBI) has made it mandatory for banks to link new floating rate loans to retail
borrowers and micro, small and medium enterprises (MSMEs) to an external benchmark. Banks can
choose one of the following benchmarks: i) RBI’s repo rate ii) Three-month treasury bill yield iii) Six-
month Treasury bill yield iv) Any other benchmark approved by Financial Benchmarks India Pvt. Ltd.
• Borrowers have often complained about the lack of transparency in the manner in which banks fix interest
rates and how the change in these rates is determined. An external benchmark-linked rate will provide
greater transparency as one can compare products across lenders by looking at which benchmark
they are using and what is the spread the banks are charging over and above the benchmark. Hence,
statement 2 is correct.
• Borrowers may also need to weigh the impact of increased volatility as the interest rate under the external
benchmark shall be reset at least once in three months. Benchmark rates, such as the RBI’s repo rate can
move down but it can also move up. Borrowers will need to be in a position to withstand any sudden
increase in rates as well. Hence, statement 3 is not correct.

Q 100.A
• The European Commission (EC) launched 'The Global coalition for biodiversity' on World Wildlife
Day 2020. The Coalition is made up of zoos, aquariums, botanical gardens, national parks, and
natural history and science museums from around the world.
• The coalition provides the opportunity to national parks, aquariums, botanic gardens, zoos, science and
natural history museums to raise public awareness about the dramatic effects of the biodiversity crisis
with their collections, education and conservation programmes,
• The Commission also encourages national, regional and local authorities, non-governmental
organisations, businesses, scientists and individual citizens to play their part in raising awareness ahead of
the United Nations Biodiversity Summit in October 2020.

Copyright © by Vision IAS


All rights are reserved. No part of this document may be reproduced, stored in a retrieval system or
transmitted in any form or by any means, electronic, mechanical, photocopying, recording or otherwise,
without prior permission of Vision IAS.

41 www.visionias.in ©Vision IAS

You might also like